Você está na página 1de 73

Human Sexuality 14Mar2009

1 – What is Human Sexuality?


1) Which of the following describes human sexuality?
a) How men and women reproduce to further the human species
b) The ways in which pleasure is derived from genital stimulation
c) The methods of communication and expression of sexual feelings
d) The ways in which humans are physically enabled to reproduce sexually
e) The ways in which we experience and express ourselves as sexual beings
Match the person with their scientific research:
2.1) Encyclopedia of sexuality between 1897 and 1910 a) Sigmund Freud
2.2) Sadomasochism, bestiality, necrophilia b) Havenlock Ellis
2.3) Sex drive is the principle motivating force c) Alfred Kinsey
2.4) Large scale sexual studies in 1930s and 1940s d) Richard von Krafft-Ebing
3) Which of the following focuses of biological sexual research would be the least
applicable to patient care?
a) Overcoming sexual problems
b) Helping infertile couples conceive
c) Hormones of sexual response
d) Sexual arousal and response
e) Drugs with sexual side-effects
4.1) Gender stereotypes are not inherent in our genetic heritage, but are acquired through
cultural expectations and socialization. Which of the following is NOT a typical male
role stereotype?
a) Dependent
b) Active
c) Strong
d) Logical
e) Hunter
4.2) Which of the following is NOT true regarding cross-cultural variations?
a) Kissing is common in most cultures
b) In most cultures, sex is more frequent among young people
c) Childhood masturbation is encouraged in most cultures
d) Monogamy is the most prevalent form of relationship worldwide
e) Polygyny is the most common form of worldwide polygamy
4.3) Which of the following describes fraternal polyandry?
a) Fraternal male twins sharing multiple female partners
b) Fraternal female twins sharing multiple male partners
c) A male with two fraternal female partners
d) A female with two fraternal male partners
e) Fraternal male twins sharing fraternal female twins
5.1) Who developed the original Psychoanalytical Theory of human sexual behavior?
a) Sigmund Freud
b) Havenlock Ellis
c) Alfred Kinsey
d) Richard von Krafft-Ebing
e) Watson and Skinner

DO NOT DISTRIBUTE -1-


Human Sexuality 14Mar2009

5.2) The Psychoanalytical Theory said the Ego was meant to shield the conscious mind
using defense mechanisms, such as forgetting traumatic events. Although sexual ideas
may be banished to unconsciousness, how may they be manifested?
a) Dreams
b) Aggression
c) Promiscuity
d) Fear
e) All of the above
5.3) Who developed the concept of erogenous zones, stating that many parts of the body
(not just genitals) are responsive to sexual stimulation?
a) Sigmund Freud
b) Havenlock Ellis
c) Alfred Kinsey
d) Richard von Krafft-Ebing
e) Watson and Skinner
5.4) The Oedipus Complex is a conflict during which stage of infant development where
the boy wishes to possess his mother sexually and perceives his father as a rival in love?
a) Oral
b) Anal
c) Phallic
d) Latency
e) Genital
5.5) Behaviorists Watson and Skinner speculated that punishing children for sexual
exploration (e.g. masturbation) would have what future results?
a) Associating sexual thoughts with feelings of guilt or anxiety
b) Associating sexual stimulation with feelings of guilt or anxiety
c) A desire to become involved dominating sexual relationships
d) A desire to seeks sexual pleasure through any means possible
e) Associating sexual acts with childhood
5.6) In the social-learning theory of human sexuality, what term is used for acquiring
knowledge and skills by observing others?
a) Plagiarizing
b) Duplicating
c) Replicating
d) Modeling
e) Copulation
6) Which of the following contributes the least to differences in sociocultural
perspectives of sexuality?
a) Age
b) Education
c) Religion
d) Ethnicity
e) Gender
7) It is important to include many perspectives in the understanding of sexuality as what
is meaningful, right, wrong, and natural vary widely between different cultures.
a) True

DO NOT DISTRIBUTE -2-


Human Sexuality 14Mar2009

b) False
8.1) Knowing enough about the biology of sex will enable someone to make the right
sexual decisions.
a) True
b) False
8.2) Some ancient Greek men took on an adolescent male as a lover and pupil.
a) True
b) False
8.3) In general, women of the ancient world were treated as personal property (chattels).
a) True
b) False
8.4) The production of illustrated sex manuals originated in modern times.
a) True
b) False
8.5) Graham crackers originated as a means for helping young men control their sexual
appetites.
a) True
b) False
8.6) The Trobriander tribe in Africa expected young boys and girls to engage in
intercourse when they first became biologically old enough.
a) True
b) False
8.7) The female redback spider eats the male redback spider immediately after sex.
a) True
b) False
8.8) Psychoanalysts may believe dreams of airplanes, bullets, snakes, sticks, and similar
objects symbolize male genitalia (phallic symbolism). Research has shown significant
evidence of this being true.
a) True
b) False
9.1) Sexual relations between a person and an animal is called:
a) Incest
b) Monogamy
c) Pederasty
d) Bestiality
e) Polygyny
9.2) Which of the following is defined as sexual intercourse between two people who are
not married?
a) Coitus
b) Copulation
c) Fornication
d) Monogamy
e) Polygamy
9.3) Incest is defined as sex between a person and:
a) An animal
b) A plant

DO NOT DISTRIBUTE -3-


Human Sexuality 14Mar2009

c) A close relative
d) A lover
e) A spouse
9.4) Achieving sexual gratification through hurting or humiliating others is called:
a) Medical school
b) Pederasty
c) Bisexual
d) Masochism
e) Sadism
9.5) Pederasty is a sexual love of:
a) Women
b) Men
c) Girls
d) Boys
e) Animals
9.6) Cunnilingus and fellatio are the acts of sexual activity involving the female and male
genitalia, respectively.
a) True
b) False

2 – Research Methods in Human Sexuality


1) Which of the following is NOT a goal of the science of human sexuality?
a) Formulating a research question
b) Framing the research question in the form of a hypothesis
c) Testing the hypothesis
d) Drawing conclusions
e) Using different species to confirm results
2.1) During Kinsey’s survey study between 1938 and 1949, a correlation between which
of the following was found?
a) Age and masturbation
b) Race and duration of intercourse
c) Education and participation in oral sex
d) Wealth and frequency of anal sex
e) Height and number of sexual partners
2.2) The results of Kinsey’s survey showed people seemed to find it difficult to describe
which of the following?
a) If they had participated in sexual activities
b) Which sexual activities they did the most
c) Which sexual positions they had done
d) The frequency of sexual activities
e) The number of sexual partners they had
3) Masters and Johnson were the first to conduct what type of sexual behavior study?
a) Dating in a public setting
b) Sexual stimulation in a laboratory setting
c) Sexual stimulation in a private setting
d) Masturbation in a public setting

DO NOT DISTRIBUTE -4-


Human Sexuality 14Mar2009

e) Interracial sexual contact in a public setting


4.1) A sexual study is being conducted in which participants are free to withdraw from
the study at any time and without penalty. Which of the following ethical research issues
does this address?
a) Exposing participants to harm
b) Confidentiality
c) Informed consent
d) Use of deception
4.2) A sexual study is being conducted in which applicants must show identification to
prove they are adults prior to becoming participants in the study. Which of the following
ethical research issues does this address?
a) Exposing participants to harm
b) Confidentiality
c) Informed consent
d) Use of deception
5.1) Some sexual researchers have engaged in “swinging” with the people they studied.
a) True
b) False
5.2) Masters and Johnson created a transparent artificial penis containing photographic
equipment to study female sexual response.
a) True
b) False
5.3) People who attend church regularly tend to be more satisfied with their relationships.
a) True
b) False
5.4) Studies have proven that Sildenafil (Viagra) causes risky sexual behavior.
a) True
b) False
5.5) There is evidence that the victims of genital mutilation (the women what are
circumcised) tend to internalize the beliefs of their culture. Not only do they generally
support female circumcision, they also accept the right of husbands to beat their wives.
a) True
b) False
5.6) Researchers publish the names of participants of sex research in professional
journals.
a) True
b) False
6.1) Anthropomorphism is defined as attributing human characteristics to:
a) Sexual objects
b) Plants
c) Animals
d) Liquids
e) Automobiles
6.2) Which of the following describes what a penile strain gauge measures?
a) How much weight the penis can hold
b) The length of the penis during sexual arousal

DO NOT DISTRIBUTE -5-


Human Sexuality 14Mar2009

c) The circumference of the penis during sexual arousal


d) The length of the penis without sexual arousal
e) The circumference of the penis without sexual arousal
6.3) A vaginal plethysmograph is a tampon-shaped probed that is inserted into the vagina,
using light reflected from the vaginal walls to measure which of the following?
a) Lymphatic flow
b) Orgasm duration
c) Orgasm strength
d) Vasocongestion
e) Anatomical relations

3 – Female Sexual Anatomy and Physiology


1.1) Which part of the female sexual anatomy consists of fatty tissue covering the pubic
bone joint to serve as a cushion during sex?
a) Mons veneris
b) Labia majora
c) Labia minora
d) Introitus
e) Clitoris
1.2) The vestibule is the area within the labia minora that contains the vaginal opening
and which of the following?
a) Clitoris
b) Labia majora
c) Mons veneris
d) Introitus
e) Urethra
1.3) Anatomical changes to which of the following areas is used in arguments of virginity
and in arguments of sexual abuse, although such evidence is controversial?
a) Clitoris
b) Labia majora
c) Mons veneris
d) Introitus
e) Urethra
2) Which of the following structures does NOT underlie the vulva?
a) Vestibular bulbs
b) Bartholin glands
c) Prepuce
d) Vaginal sphincters
e) Anal sphincters
3) Which of the following is the only female sex organ used specifically for pleasure and
contains corpora cavernosa?
a) Mons veneris
b) Hymen
c) Labia minora
d) Introitus
e) Clitoris

DO NOT DISTRIBUTE -6-


Human Sexuality 14Mar2009

4.1) Female genital cutting (FGC), also known as female genital mutilation (FGM) or
female circumcision, refers to "all procedures involving partial or total removal of the
external female genitalia or other injury to the female genital organs whether for cultural,
religious or other non-therapeutic reasons." The World Health Organization defines
infibulation (Type III FGC) as which of the following?
a) Removal of the clitoris (clitoridectomy)
b) Removal of the labia minora
c) Surgical closure of the labia majora
d) All of the above
4.2) Infibulation is the most common in which of the following African areas?
a) Somalia, Egypt, Sudan
b) Libya, Algeria, Morocco
c) Niger, Chad, Nigeria
d) Angola, Zambia, Namibia
e) Madagascar, Mozambique, South Africa
4.3) Which of the following is NOT a common complication of FGC/FGM procedures?
a) Future sexual dysfunction
b) Infertility
c) Bleeding
d) Edema
e) Infection
5) The condition of the hymen can be used to prove virginity; that is, an intact hymen
means the female has not had penetrating sex.
a) True
b) False
6.1) The opening in the middle of the cervix leading to the uterus is called the:
a) Os
b) Hole
c) Slit
d) Pouch
e) Introitus
6.2) Which of the following is the upper end of the vagina whose secretions contribute to
the chemical balance of the vagina?
a) Ovaries
b) Uterus
c) Fallopian tubes
d) Cervix
e) Bartholin glands
7.1) Which of the following pathologies is characterized by menstrual pain?
a) Vaginitis
b) Cervical cancer
c) Endometriosis
d) Ectopic pregnancy
e) Ovarian cyst
7.2) Which of the following pathologies can be most easily prevented with proper
hygiene?

DO NOT DISTRIBUTE -7-


Human Sexuality 14Mar2009

a) Vaginitis
b) Cervical cancer
c) Endometriosis
d) Ectopic pregnancy
e) Ovarian cyst
8) Which of the following is NOT part of the routine pelvic exam?
a) Inspection of the external genitalia for swelling and irritation
b) Inspection of the internal genitalia with a speculum
c) Bimanual exam with two fingers in the vagina and other hand on the abdomen
d) Rectovaginal inspection with a finger in the vagina and a finger in the anus
e) Prostate exam with two fingers in the anus
9) Menopause is the cessation of menstruation that generally takes place in women ages
46-50. The associated falling off of production of hormones causes all of the following
menopause symptoms EXCEPT:
a) Hot flashes
b) Joint pain
c) Night sweats
d) Cold sweats
e) Weight gain
10) Although there is a social-religious stigma behind coitus during menstruation,
contraction of the uterus during orgasm may help relieve cramping and helps to clear the
lining and clotted blood left inside the uterus.
a) True
b) False
11.1) Dysmenorrhea (pain during menstruation) is very common in women. Which of the
following is NOT a symptom associated with menstruation?
a) Bloating
b) Nausea
c) Headaches
d) Fever
e) Cramps
11.2) Elevated levels of prostaglandins during menstruation is associated with which of
the following symptoms?
a) Bloating
b) Nausea
c) Headaches
d) Fever
e) Cramps
11.3) Psychologically, women need recognize that there is nothing wrong with them
during dysmenorrhea, this a naturally occurring event and they are blameless for their
physical discomfort.
a) True
b) False
12.1) Pudendum is derived from Latin meaning “something to be ashamed of.”
a) True
b) False

DO NOT DISTRIBUTE -8-


Human Sexuality 14Mar2009

12.2) Many males (and even some females) believe erroneously that for women urination
and coitus occur through the same bodily opening.
a) True
b) False
12.3) Breast size helps determine the quantity of milk that can be produced.
a) True
b) False
12.4) Abortions and silicone breast implants have been shown to increase a woman’s risk
of breast cancer.
a) True
b) False
12.5) Monthly breast self-examinations (BSEs) are recommended by the American
Cancer Society.
a) True
b) False
12.6) Many women have hot flashes at menopause.
a) True
b) False
13.1) The circular area of colored skin surrounding the nipple is called the:
a) Areola
b) Bartholin gland
c) Crura
d) Os
e) Prepuce
13.2) An operation to remove the uterus but not the cervical stump is called a(n):
a) Amenorrhea
b) Episiotomy
c) Hysterectomy
d) Lumpectomy
e) Mastectomy
13.3) Climacteric is a long-term process synonymous with menopause, or a phase in a
man’s life corresponding to female menopause.
a) True
b) False
13.4) Anorexia nervosa is an eating disorder characterized by spells of excessive eating.
a) True
b) False
13.5) The clitoral crura are an internal portion of the clitoris that are shaped like an
inverted "V" with the vertex of the "V" connecting to the clitoral body. They are near the
vestibular bulbs, also known as the clitoral bulbs.
a) True
b) False
13.6) Surgical removal of the entire breast is known as a(n):
a) Amenorrhea
b) Episiotomy
c) Hysterectomy

DO NOT DISTRIBUTE -9-


Human Sexuality 14Mar2009

d) Lumpectomy
e) Mastectomy
13.7) A fibroadenoma is a:
a) Saclike structure filled with fluid or diseased material
b) A tumor that is lethal or likely to cause harm
c) Pain in the breast
d) A benign, fibrous tumor
e) Surgical removal of a lump from the breast
13.8) A surgical incision in the perineum that may be made during childbirth to protect
the vagina from tearing (although the prevalence of this surgery is declining) is known as:
a) Amenorrhea
b) Episiotomy
c) Hysterectomy
d) Lumpectomy
e) Mastectomy
13.9) The internal female genitals are known as the pudendum.
a) True
b) False
13.10) The prepuce is the fold of skin covering the glans of the clitoris and is also known
as the clitoral hood.
a) True
b) False
13.11) Derived from the French term “shower”, this device is meant to clean the vagina
but has been associated with long term complications such as yeast infections.
a) Dildo
b) Speculum
c) Pap test
d) Tampon
e) Douche
13.12) Cramps or painful menstruation associated with a disease process is known as:
a) Primary amenorrhea
b) Primary dysmenorrhea
c) Secondary amenorrhea
d) Secondary dysmenorrhea
e) Menarche
13.13) Which of the following is often caused by hormonal disturbances from the
hypothalamus (body temperature, motivation, emotion) and the pituitary gland (prolactin,
oxytocin, growth hormone) or from premature menopause, or intrauterine scar formation.
a) Primary amenorrhea
b) Primary dysmenorrhea
c) Secondary amenorrhea
d) Secondary dysmenorrhea
e) Menarche
13.14) Growth and inflammation outside of the inner membrane of the uterus is:
a) Cystitis
b) Endometriosis

DO NOT DISTRIBUTE - 10 -
Human Sexuality 14Mar2009

c) Osteoporosis
d) Vaginitis
e) Mittelschmerz
13.15) Traits that distinguish women from men but are not directly involved in
reproduction are known as secondary sex characteristics.
a) True
b) False
13.16) The first menstration is known as:
a) Primary amenorrhea
b) Primary dysmenorrhea
c) Secondary amenorrhea
d) Secondary dysmenorrhea
e) Menarche
13.17) Ovulation pain is known as:
a) Cystitis
b) Endometriosis
c) Osteoporosis
d) Vaginitis
e) Mittelschmerz
13.18) The Papanikolaou test (Pap test) is a medical screening method primarily designed
to detect premalignant and malignant processes in the ectocervix.
a) True
b) False
Match the definition with the term:
13.19) Hypothalamus hormone that causes pituitary gonadotropin release a) Prolactin
13.20) Stimulates the production of milk b) Oxytocin
13.21) Stimulates follicle development c) Follicle-stimulating hormone (FSH)
13.22) Stimulates uterine contractions and milk ejection d) Luteinizing hormone (LH)
13.23) Triggers ovulation e) Gonadotropin-releasing hormone (Gn-RH)
13.24) Estrus refers to the phase in the estrous cycle when the woman is sexually
receptive (e.g. “in heat”) and may have associated traits such as the lordotic reflex.
a) True
b) False
13.25) The first phase of the menstrual cycle that lasts about nine or ten days is known as:
a) Ovulation
b) Pituitary
c) Premenstrual syndrome (PMS)
d) Proliferation
e) Secretory
13.26) A premenstrual phase characterized by severe changes in mood and impairment of
functions at work, at school, or in social relationships is known as:
a) Premenstrual syndrome (PMS)
b) Premenstrual dysphoric disorder (PMDD)
c) Anorexia nervosa
d) Mastalgia
e) Hysteria

DO NOT DISTRIBUTE - 11 -
Human Sexuality 14Mar2009

14.1) Which positions is NOT a recommended position for a breast self exam (BSE)?
a) In the shower
b) Standing in front of a mirror
c) Lying down
d) During sexual activity
14.2) Which of the following is NOT true regarding menopause?
a) Menopause is normal
b) Menopause is not considered a disease
c) After menopause, women need complete replacement of estrogen
d) Menopause is not always accompanied with depression and anxiety
e) Not all women experience hot flashes at menopause
f) A woman who has had a hysterectomy may still undergo menopause
g) Menopause does not end a woman’s sexual appetite
h) A woman’s general level may be higher after menopause

4 – Male Sexual Anatomy and Physiology


1.1) Which of the following areas of the penis would be the least sensitive?
a) Glans
b) Corona
c) Frenulum
d) Crura
1.2) The penis contains which of the following types of cylinders?
a) 1 Corpus cavernosum, 1 Corpus spongiusum
b) 1 Corpus cavernosum, 2 Corpora spongiosa
c) 2 Corpora cavernosa, 1 Corpus spongiosum
d) 2 Corpora cavernosa, 2 Corpora spongiosa
e) 3 Corpora cavernosa, 3 Corpora spongiosa
2) Which of the following is NOT true regarding male circumcision?
a) It involves the removal of the prepuce
b) Some believe circumcision improves hygiene and removes smegma (a cheese-
like, foul smelling secretion that may accumulate)
c) Circumcision is least common amongst European American men
d) Research has shown circumcision can reduce the spreading of HIV
e) Circumcision can lessen sexual sensation
f) The American Academy of Pediatrics does not recommend circumcision as a
universal procedure to prevent sexually transmitted disease
3.1) Zilbergeld (1999) reported that women complained about their partner’s penis size
more than other relationship issues such as communication or emotional atmosphere.
a) True
b) False
3.2) Which of the following would NOT reduce penis size?
a) Cold air
b) Cold water
c) Fear
d) Anxiety
e) Sexual excitement

DO NOT DISTRIBUTE - 12 -
Human Sexuality 14Mar2009

4) Which of the following raises or lowers the testicles in the scrotum in an attempt to
keep testicular temperature about 5 to 6 degrees below body temperature (98.6ºF)?
a) Dartos muscle
b) Spermatic cord
c) Vas deferens
d) Cremaster muscle
e) Corpus cavernosa
5.1) Which of the following is the correct route of sperm?
a) Seminiferous tubules, epididymis, Vas deferens, ejaculatory duct, urethra
b) Seminiferous tubules, Vas deferens, epididymis, ejaculatory duct, urethra
c) Seminiferous tubules, Vas deferens, ejaculatory duct, epididymis, urethra
d) Seminiferous tubules, ejaculatory duct, urethra, epididymis, Vas deferens
e) Epididymis, Seminiferous tubules, Vas deferens, ejaculatory duct, urethra
5.2) The fluid produced by which of the following is rich in fructose, which nourishes the
sperm, and secretes 70% of the accessory fluid for ejaculation?
a) Prostate
b) Bulbourethral gland
c) Seminal vesicle
d) Epididymis
e) Seminiferous tubules
6) Cancer of the testicles is a rare form of cancer (1% of all new cancers in men) and
between the ages of 20-34 accounts for what percentage of cancer deaths?
a) 0.5%
b) 1%
c) 2%
d) 5%
e) 10%
7) Testicular self-exams should be done regularly by rolling each testicle between the
thumb and fingers. Which of the following is NOT a warning sign during self-exam per
the National Cancer Institute?
a) Slight enlargement of one of the testes
b) Change in consistency of the testicle
c) Dull aches in the lower abdomen or groin
d) Sensation of dragging or heaviness in the testicles
e) Burning sensation during urination
8.1) What percentage of men over the age of 50 will develop benign prostatic hyperplasia
(BPH), which can be treated with alpha blockers?
a) 10%
b) 20%
c) 50%
d) 70%
e) 80%
8.2) How many men will develop prostate cancer, which is life-threatening, in their life?
a) 1 in 8
b) 1 in 6
c) 1 in 4

DO NOT DISTRIBUTE - 13 -
Human Sexuality 14Mar2009

d) 1 in 2
9.1) Men have nocturnal erections every 90 minutes or so as they sleep, generally
occurring during rapid eye movement (REM).
a) True
b) False
9.2) Men must make a conscious effort to become aroused (erect) during sexual touching.
a) True
b) False
10) Because of a lack of nerve connection between genital organs and the brain, men with
spinal injuries can achieve erection sand ejaculate in response to direct tactile stimulation
of the penis even if they have no sensation.
a) True
b) False
11) What division of the nervous system is involved in male erection?
a) Somatic afferent system
b) Somatic efferent system
c) Autonomic (both branches)
d) Autonomic sympathetic system
e) Autonomic parasympathetic system
12.1) Peyronie disease is a disorder involving:
a) Overgrowth of the labia minora
b) Oversize clitoris
c) Overgrowth of the penile prepuce
d) Excessive curvature of the penis
e) An erection lasting for hours
12.2) Priapism is a condition described as:
a) Overgrowth of the labia minora
b) Oversize clitoris
c) Overgrowth of the penile prepuce
d) Excessive curvature of the penis
e) An erection lasting for hours
12.3) In retrograde ejaculation, semen is ejected into the bladder.
a) True
b) False
13.1) During the expulsion stage of ejaculation, which of the following is most involved
in movement of semen?
a) Prostate
b) Seminal vesicles
c) Ampulla of Vas Deferens
d) Muscles at the base of the penis
e) Urethral bulb
13.2) Which center(s) in the spinal cord regulate ejaculation?
a) Sacral
b) Lower lumber
c) Higher lumbar
d) Sacral and lower lumber

DO NOT DISTRIBUTE - 14 -
Human Sexuality 14Mar2009

e) Sacral and higher lumber


14.1) The penis is made of bone and muscle.
a) True
b) False
14.2) The father determines a child’s gender.
a) True
b) False
14.3) Sperm of the fruit fly (drosophila) are longer than human sperm.
a) True
b) False
14.4) Morning erections indicate the need to urinate.
a) True
b) False
14.5) Men can will themselves to have an erection.
a) True
b) False
14.6) The penis has a mind of its own.
a) True
b) False
14.7) Men can orgasm without ejaculation.
a) True
b) False
15.1) A condition in which the foreskin is so tight it cannot be withdrawn from the glans
penis is known as:
a) Cryptorchidism
b) Circumcision
c) Phimosis
d) Paraplegia
e) Urethritis
15.2) What is secreted by interstitial (Leydig) cells?
a) Sperm
b) Estrogen
c) Testosterone
d) Alkalizing fluid for the ejaculate
e) Fructose for the ejaculate
15.3) Androgens are male sex hormones.
a) True
b) False, they are female sex hormones
15.4) After a vasectomy (ligation of Vas Deferens), how long should the patient wait
before having sex again?
a) A couple days
b) A couple weeks
c) Three months
d) Six months
e) One year
16.1) Prostate cancer progresses slowly and is usually detected by which means?

DO NOT DISTRIBUTE - 15 -
Human Sexuality 14Mar2009

a) Digital rectal exam (DRE)


b) Prostate specific antigen (PSA)
c) CT scan
d) MRI scan
e) All of the above
16.2) What groups ae at increased risk for prostate cancer?
a) European American men with no family history
b) European American men with a family history
c) African American men with no family history
d) African American men with a family history
17) Koro-Genital Reaction Syndrome is a disorder found in some Asian men who believe
their penis will shrink and retract into their body even though it is not anatomically
possible, such as during attempts to urinate or when it becomes could outside.
a) True
b) False
18) Prescriptions for testosterone replacement therapy has shown conclusive results.
a) True
b) False

5 – Sexual Arousal and Response


1) When it comes to sexual arousal, people tend to be:
a) Taste oriented
b) Smell oriented
c) Sound oriented
d) Sight oriented
2) Smell may also indicate preference of sexual partners. In one study it was found that
gay men preferred the axillary odors of gay men over heterosexuals or lesbians. This
same study found that lesbians and heterosexuals preferred the axillary odors of
heterosexual men and lesbians to that of gay men.
a) True
b) False
3) Which of the following is NOT a primary erogenous zone?
a) Genitals
b) Outer thigh
c) Perineum
d) Anus
e) Breasts
4) Which of the following play a more minor role in sexual arousal?
a) Sight and touch
b) Hearing and sight
c) Hearing, taste, and smell
d) Touch and smell
e) Taste, sight, and touch
5.1) Any sexual arousal from non-drug aphrodisiacs is most likely a result of the placebo
effect as they have not been shown to be effective.
a) True

DO NOT DISTRIBUTE - 16 -
Human Sexuality 14Mar2009

b) False
5.2) Which of the following is likely the most potent aphrodisiac?
a) L-dopa
b) Arginine
c) Amyl nitrate
d) Testosterone
e) Nitroglycerine
6) Which of the following drugs enhances sexual arousal in males and females?
a) Marijuana
b) Alcohol
c) Cocaine
d) Amphetamines
e) Hallucinogens
7) The cerebral cortex is involved in sexual ____ and the limbic system is involved in
sexual ____.
a) Arousal; Arousal
b) Arousal; Thoughts
c) Thoughts; Arousal
d) Thoughts; Thoughts
8) Which of the following hormones has been shown to be the dominating factor in the
sex drive of both men and women, and may be involved in organizing the fetal brain
toward a masculine or feminine direction.
a) Estrogen
b) Progesterone
c) Testosterone
d) Oxytocin
e) Leutinizing hormone
9) Which stage of sexual response (Masters and Johnson) is associated with
vasocongestion and increased lubrication?
a) Excitement phase
b) Plateau phase
c) Orgasmic phase
d) Resolution phase
10) Unlike the four-stage model of sexual response, the three-stage model by Kaplan
does not have a specific order and is based mainly on:
a) Orgasm
b) The man
c) The woman
d) Sexual dysfunction
e) The lunar calendar
11) Which of the following is true of multiple orgasms?
a) Men can have them and women cannot
b) Women can have them and men cannot
c) Men and women can have them
d) Neither men nor women can have them

DO NOT DISTRIBUTE - 17 -
Human Sexuality 14Mar2009

12) Although Masters and Johnson felt there was one type of orgasm that men and
women had, Singer and Singer proposed there were:
a) No definite number of types
b) Two types
c) Three types
d) Four types
e) Five types
13) The Grafenberg spot (G-spot) is anatomically located 1 to 2 inches from the vaginal
entrance on what wall?
a) Lateral
b) Anterior
c) Posterior
d) Encircling
14.1) Menstrual cycles of women who live together tend to become synchronized.
a) True
b) False
14.2) The brain is considered an erogenous zone.
a) True
b) False
14.3) Spanish fly (cantharidin) inflames the urethra causing a burning sensation, which
can be misinterpreted as a sexual feeling.
a) True
b) False
14.4) In a study by Proctor (1974), written descriptions of male and female orgasms
could not be told apart.
a) True
b) False
14.5) Masters and Johnson (1966) found that orgasms during masturbation were more
physiologically intense than those experienced during intercourse.
a) True
b) False
15.1) Sex flush is a reddish rash that appears on what part of the body during the
excitement phase of the sexual response cycle?
a) Labia
b) Face
c) Chest
d) Hands
e) Buttocks
15.2) Sex skin refers to reddening of what part of the body during the plateau phase of the
sexual response cycle?
a) Labia
b) Face
c) Chest
d) Hands
e) Buttocks

DO NOT DISTRIBUTE - 18 -
Human Sexuality 14Mar2009

16.1) Studies have shown the perspiration (e.g. underarm secretions) may make people
more sexually attractive.
a) True
b) False
16.2) Which of the following is considered a date-rape drug?
a) AZT
b) Metoprolol
c) HCTZ
d) Cialis
e) Rohypnol

6 – Gender Identity and Gender Roles


1.1) Which chromosome pair determines zygotic sex?
a) 13
b) 19
c) 21
d) 23
1.2) Each male (Wolffian) duct develops into:
a) Epididymis
b) Vas Deferens
c) Seminal vesicle
d) All of the above
2) Gender dysphoria refers to:
a) Pre-natal exposure to testosterone
b) Being born homosexual
c) Having homosexual tendencies
d) Elation with one’s gender
e) Incongruity between anatomic gender and gender identity
Match the following terms with their definition:
3.1) A person with a strong desire to be the opposite sex a) Hermaphroditism
3.2) Having gonads of one sex and genitalia of the opposite sex b) Intersexualism
3.3) Having ovarian and testicular tissue c) Transsexualism
3.4) Male-to-female surgical sex changes can be done more precisely than female-to-
male (phalloplasty) sex changes.
a) True
b) False
4.1) Without male sex hormones, female external genitalia would develop.
a) True
b) False
4.2) 17 out of 18 boys who appeared to have female external sex organs suddenly
developed male sex organs at puberty, when male sex hormones went to work. These
boys were diagnosed with Dominican Republic syndrome, a genetic enzyme disorder that
prevents testosterone form masculinizing the external genitals.
a) True
b) False
4.3) The sex of a crocodile depends on the temperature at which eggs develop.

DO NOT DISTRIBUTE - 19 -
Human Sexuality 14Mar2009

a) True
b) False
4.4) Sex reassignment surgery can implant internal genitalia and internal reproductive
organs.
a) True
b) False
5.1) Autogynephilic refers to:
a) Females who are sexually stimulated by fantasies of their bodies being male
b) Females who are sexually stimulated by fantasies of their bodies
c) Males who are sexually stimulated by fantasies of their bodies being female
d) Males who are sexually stimulated by fantasies of their bodies
5.2) Congenital adrenal hyperplasia is the most common form of intersexualism and
involves:
a) Female internal genitalia and male external genitalia
b) No internal genitalia and male external genitalia
c) Male internal genitalia and female external genitalia
d) No internal genitalia and female external genitalia
5.3) A patient is born with karotype 46XY but has no problem with production of
testosterone or mullarian inhibiting hormone. Their vagina ends in a blind pouch and
testes are lodged in the inguinal canal. Which of the following is the best description of
this patient:
a) Male pseudo-hermaphrodite
b) Female pseudo-hermaphrodite
c) Androgen insensitivity syndrome
d) Turner’s syndrome
e) Klinefelter syndrome
5.4) A patient is born as a male with karotype 47XXY and does not develop appropriate
male secondary sex characteristic. Which of the following is the best description of this
patient:
a) Male pseudo-hermaphrodite
b) Female pseudo-hermaphrodite
c) Androgen insensitivity syndrome
d) Turner’s syndrome
e) Klinefelter syndrome
6) Gender identity disorder (GID) is a condition in which a person has been born one
gender, usually on the basis of their sex at birth, but identifies as belonging to another
gender, and feels significant discomfort or the inability to deal with this condition.
a) True
b) False

7 – Attraction and Love-Binding Forces


Not Covered

8 – Relationships and Communications


Not covered

DO NOT DISTRIBUTE - 20 -
Human Sexuality 14Mar2009

9 – Sexual Techniques and Behavior Patterns


1.1) Early Judeo-Christians viewed masturbation as:
a) A sin
b) Something to be ignored
c) A natural act
d) A disgrace to the family
e) Symbolism of God
1.2) Many people, such as Slyvester Graham (Graham cracker), Dr. J. J. Kellogg
(cereal), Benjamin Rush, Krafft-Ebring, and others, believe that masturbation was
associated with disease processes.
a) True
b) False
2) Which of the following is NOT true regarding masturbation?
a) Men are more likely to masturbate than women
b) Married people are less likely to masturbate
c) People with more education masturbate more frequently
d) African Americans are the most likely ethnicity to masturbate
3.1) During sexual activity, women are more likely to fantasize about past sexual partners
than men.
a) True
b) False
3.2) One study found that ____ of men and ____ of women fantasize about someone
other than their sex partner.
a) 98%; 80%
b) 52%; 40%
c) 40: 52%
d) 80%; 98%
4) Which of the following is NOT a common foreplay technique?
a) Touching erogenous zones
b) Genital stimulation
c) Breast stimulation
d) Kissing
e) Vaginal insertion
5) Which ethnicity is the most likely to engage in oral-genital stimulation?
a) Asian Americans
b) South Americans
c) European Americans
d) African Americans
e) Australians
6) More highly educated individuals are more likely to have practiced oral sex.
a) True
b) False
7) Which of the following is the most frequent reason for abstaining from oral sex?
a) Odor
b) Taste
c) Smell

DO NOT DISTRIBUTE - 21 -
Human Sexuality 14Mar2009

d) Texture
e) Shyness and embarrassment
8.1) Which of the following sexual positions is NOT recommended for pregnant women?
a) Male superior
b) Female superior
c) Lateral entry
d) Rear entry
8.2) Which position can be awkward for penile entry and may be difficult in achieving
deep penetration?
a) Male superior
b) Female superior
c) Lateral entry
d) Rear entry
8.3) Which position is most associated with queefing (vaginal flatulence) after sex?
a) Male superior
b) Female superior
c) Lateral entry
d) Rear entry
9) Coital fantasies allow couples to inject sexual variety without being unfaithful, and
there does not appear to be any connection between sexual dissatisfaction with one’s
relationship and coital fantasies.
a) True
b) False
10.1) More highly educated individuals are more likely to have practiced anal sex.
a) True
b) False
10.2) Which of the following should be used with anal sex to help prevent STDs?
a) Spermicidal lubricant
b) Water-based lubricant
c) Petroleum jelly
d) Condoms
e) The pull-out method
11.1) Married people rarely, if ever, masturbate.
a) True
b) False
11.2) European American men are more likely to masturbate than African American men.
a) True
b) False
11.3) Women who refrained from masturbation during adolescence are more likely to
find gratification in marital coitus.
a) True
b) False
11.4) Women achieve orgasm more reliably through coitus than masturbation.
a) True
b) False
11.5) Most women masturbate by inserting a finger or other object into the vagina.

DO NOT DISTRIBUTE - 22 -
Human Sexuality 14Mar2009

a) True
b) False
11.6) Heterosexual people do not fantasize about sexual activity with people of their own
gender.
a) True
b) False
11.7) Statistically speaking, oral sex is the norm for today’s young married couples.
a) True
b) False
11.8) When lovers fantasize about other people, their relationship is in trouble.
a) True
b) False
11.9) Anal sex is more common among less educated people.
a) True
b) False
12.1) Coitus interruptus refers to:
a) Using condoms for sexual intercourse
b) Stopping coitus due to bodily noises
c) Pulling out during coitus prior to male orgasm
d) Premature ejaculation during coitus
12.2) Anilingus refers to:
a) Marriage without oral sex
b) A sexual encounter not involving oral sex
c) Anal sex with oral sex involving female genitalia
d) Anal-oral sex

10 – Sexual Orientation
1) One’s gender identity is where their sexual attraction and romantic interest lie.
a) True
b) False
2.1) On the Kinsey continuum of sexual orientation, what does a score or category of six
(6) designate?
a) Bisexual male
b) Bisexual female
c) Exclusively heterosexual
d) Exclusively bisexual
e) Exclusively homosexual
2.2) In the Storm two-dimensional model of sexual orientation, who would be described
as being high in both dimensions?
a) Asexuals
b) Heterosexuals
c) Homosexuals
d) Bisexuals
3.1) A female individual who was equally attracted to both men and women would be
called:
a) Bi-gay

DO NOT DISTRIBUTE - 23 -
Human Sexuality 14Mar2009

b) Bi-straight
c) Bi-bi
d) Biphobic
e) Heterosexual
3.2) Bisexuals are more accepted socially than homosexuals.
a) True
b) False
4) Which of the following is NOT true regarding historical and religious views?
a) In Ancient Greece, men frequently formed relationships with adolescent males
b) The Romans described feminine gay men who dressed flamboyantly and
searched certain neighborhoods for partners
c) In 15th Century Florence, male-male sexual activity was a sin
d) In the Book of Leviticus, male-male sexual activity was recommended
e) In most of Western Europe, there were penalties for non-procreative sexual acts
5.1) Many preliterate societies viewed male-male sexual interactions as normal.
a) True
b) False
5.2) Over half of American adults believe that homosexuality is morally wrong.
a) True
b) False
5.3) Among Sambian people of New Guinea, 7-10 year old males leave their parent’s
house to undergo a sexual rites passage, which includes ingestion of semen, which is
believed to give rise to puberty.
a) True
b) False
6) People who have a strong stake in maintaining stereotypical gender roles (e.g. using
derogatory names, taunting, barring gay people from events) may feel more readily
threatened by the existence of the gay male or lesbian sexual orientation, because gay
people appear to confuse or reverse those roles.
a) True
b) False
7) The 1986 Supreme Court decision Hardwick vs. Bowers, which was struck down in
2003, let stand what Georgia sodomy law?
a) Male-male public relations, such as kissing, was punishable by up to 20 years
in prison
b) Male-male sexual contact was punishable by up to 20 years in prison
c) Oral-genital or anal-genital sexual contact was punishable by up to 20 years in
prison
d) Male-male oral-genital sexual contact was punishable by up to 20 years in
prison
e) Male-male anal-genital sexual contact was punishable by up to 20 years in
prison
8) Which of the following statements is true regarding sexual orientation?
a) In twin studies, identical twins (monozygotic) were not found to be concordant
for gay male sexual orientation when compared to non-twins

DO NOT DISTRIBUTE - 24 -
Human Sexuality 14Mar2009

b) It has been proven that no X chromosomal region predisposes men to gay male
sexual orientation
c) The third interstitial nucleus of the anterior hypothalamus is smaller in gay
men than in heterosexual men
d) Genitals of gay people do not differentiate prenatally in accord with their
chromosomal sex as other factors modify this differentiation
e) Research has shown sexual orientation with levels of male or female sex
hormone in adulthood
9.1) Sigmund Freud felt gay male or lesbian sexual orientation resulted from failure to
resolve the Oedipus complex.
a) True
b) False
9.2) The majority of gay males and lesbians were not aware of sexual interest in people
of their own sex before they had sexual encounters with them.
a) True
b) False
10) Failure to form the gender-role stereotype that is consistent with one’s anatomic sex
is also known as:
a) Being a “tom boy”
b) Lesbianism
c) Gender confusion
d) Gender reversal
e) Gender nonconformity
11.1) Homosexuals have a lower likelihood of suicide than heterosexuals.
a) True
b) False
11.2) Therapy for homosexual men and women who wished to be heterosexual were:
a) Relatively successful
b) Neither successful nor unsuccessful
c) Relatively unsuccessful
d) Completely failures
12) Which of the following is considered the first step in the “coming out” process?
a) Disclosure of one’s sexual orientation to other people
b) Sexual contact with members of the same sex
c) Self-labeling as gay or lesbian
d) Attraction to members of the same sex
13.1) 79% of gay males reported having sex with strangers and 84% reported having at
least how many sexual partners in their lifetime?
a) 10
b) 20
c) 30
d) 40
e) 50
13.2) Lesbians are much less likely to have sex with strangers and have a significant
number of sexual partners.
a) True

DO NOT DISTRIBUTE - 25 -
Human Sexuality 14Mar2009

b) False
14) Gay men are more likely to have committed, affectionate relationships than lesbians.
a) True
b) False
15.1) Gay males and lesbians would prefer to be members of the other sex.
a) True
b) False
15.2) Members of ethnic minority groups in the United States are less tolerant of
homosexuals than European Americans.
a) True
b) False
15.3) Today, the majority of Americans oppose gay marriages.
a) True
b) False
15.4) A majority of Americans believe that gay people choose to be gay.
a) True
b) False
15.5) The American Psychiatric Association considers homosexuality to be a mental
disorder.
a) True
b) False
15.6) Many gay couples have lifestyles similar to those of committed heterosexual
couples and are well adjusted.
a) True
b) False
16.1) Activating effects influence sex drive and sexual orientation.
a) True
b) False
16.2) The association male-female intercourse with fear of castration (castration anxiety)
has been scientifically demonstrated.
a) True
b) False
16.3) The term femme refers to:
a) A gay male who is masculine
b) A gay male who is feminine
c) A lesbian who is masculine
d) A lesbian who is feminine
e) A straight male who is very weak

11 – Conception, Pregnancy, and Childbirth


1) Which of the following methods of optimizing the chances of conception is the easiest
for couples to try?
a) Using a basal body temperature chart
b) Analyzing urine and saliva for leutenizing hormone (LH)
c) Tracking vaginal mucus
d) Modifying sexual position

DO NOT DISTRIBUTE - 26 -
Human Sexuality 14Mar2009

2) Which of the following methods to select offspring gender is the most successful?
a) Shettler approach
b) Sperm separation
c) Body temperature
d) Modifying sexual position
e) Preimplantation genetic diagnosis
3.1) Low sperm count, which is normally 40-150 million sperm per milliliter, is the most
common reason for male infertility. Which of the following would cause low sperm
count?
a) Wearing loose underwear such as boxer briefs
b) Exercising 30 minutes a day, 3 days a week
c) Eating a well-balanced diet, including meat and dairy
d) Engaging in sexual activity multiple times a day, most days of the week
e) Spending 3 days a week relaxing in a hot tub
3.2) A female presents with infertility issues. It is found that one of her ovarian tubes is
blocked. Which of the following is the most likely cause?
a) Malformation of the Mullerian ducts
b) Diabetes
c) Endometriosis
d) Recurrent urinary tract infections (UTIs)
e) Increased hormone levels of estrogen and progesterone
4.1) A male has very low sperm count and wishes to have offspring with his wife. Which
of the following is the most reliable method for this couple?
a) Gamete intrafallopian transfer (GIFT)
b) Zygote intrafallopian transfer (ZIFT)
c) Donor in vitro fertilization
d) Embryonic transfer
e) Intracytoplasmic sperm injection (ICSI)
4.2) Which of the following methods involves combining sperm and ova in a laboratory
dish prior to implantation?
a) Gamete intrafallopian transfer (GIFT)
b) Zygote intrafallopian transfer (ZIFT)
c) Donor in vitro fertilization (Donor IVF)
d) Embryonic transfer
e) Intracytoplasmic sperm injection (ICSI)
5) Which of the following is true of sex during pregnancy?
a) Many women will show a decline in sexual interest during the first trimester
b) Sex can occur during the first trimester even if there is a risk of bleeding
c) The male-superior position is recommended during second trimester
d) There are no restrictions on sex during the third trimester
e) It is recommended to have sex if pregnant and menstruating
6) How long after pregnancy should a couple wait before continuing their sexual activity?
a) Two weeks
b) One month
c) Six weeks
d) Two months

DO NOT DISTRIBUTE - 27 -
Human Sexuality 14Mar2009

e) Three months
7.1) Prolonged athletic activity may decrease fertility in males.
a) True
b) False
7.2) A “test tube baby” is grown in a laboratory disk throughout the nine months of
gestation.
a) True
b) False
7.3) There is an all-female species of lizard that lays unfertilized eggs that develop into
identical females generation after generation.
a) True
b) False
7.4) Morning sickness is a sign that pregnancy is progressing normally.
a) True
b) False
7.5) For the first week of conception, a fertilized egg cell is not attached to its mother’s
body.
a) True
b) False
7.6) Pregnant women can have an alcoholic beverage a day without harming their babies.
a) True
b) False
7.7) The way that the umbilical cord is cut determines whether the baby will have an
“inny” or an “outy.”
a) True
b) False
7.8) In the United states, nearly one in four births are done by cesarean section.
a) True
b) False
8) Which of the following would be best for a couple who wishes to have children but the
female does not produce ova?
a) Gamete intrafallopian transfer (GIFT)
b) Zygote intrafallopian transfer (ZIFT)
c) Donor in vitro fertilization (Donor IVF)
d) Embryonic transfer
e) Intracytoplasmic sperm injection (ICSI)

12 – Contraception and Abortion


1) Which of the following methods of contraception was NOT used in the past?
a) Dung soaked in sour milk
b) Animal intestine condoms
c) Wine and garlic douche
d) Coitus interruptus (withdrawal method)
e) Holy water on the belly and prayer
2.1) What is the role of progesterone in birth control pills?
a) Decreases FSH so follicles mature more quickly

DO NOT DISTRIBUTE - 28 -
Human Sexuality 14Mar2009

b) Increases FSH so follicles do not mature


c) Decreases LH so ovulation is halted
d) Increases LH so ovulation occurs quickly
e) Causes muscles to contract and close the ovarian tubes
2.2) Which of the following is NOT true regarding birth control pills?
a) They have failure rates are below 3%
b) They increase the risk of blood clots
c) They reduce the effectiveness of certain antibiotics
d) They help prevent STDs
e) They can help reduce the risk of pelvic inflammatory disease
3) Which of the following contraception methods, which was discontinued in the United
States, involves inserting rods into the upper arm, which release progestin into the
bloodstream for five years?
a) RU486 (Mifepristone)
b) Norplant
c) Depo-provera (DMPA)
d) Nuva ring
e) IUD
4) Which of the following is the most serious risk of intrauterine devices (IUDs)?
a) Cramping
b) Uterine perforation
c) Pelvic inflammatory disease
d) Abnormal bleeding
e) Painful insertion
5) Which of the following has the highest contraceptive failure rate?
a) Birth control minipill
b) Birth control multiphasic pill
c) Norplant
d) IUD
e) Diaphragm
6) How long prior to intercourse should spermacides be applied?
a) 45-minutes
b) 30-minutes
c) 15-minutes
d) 10-minutes
e) 5-minutes
7) Why was the contraceptive sponge taken off the market in the United States in 1994?
a) High incidence of failure
b) Extreme difficulty in insertion
c) Incidence of getting stuck in the cervix
d) Bacterial contamination
e) Penile injury during sexual intercourse
8) The cervical cap contraceptive is like a diaphragm but must be fitted by a clinician.
a) True
b) False
9) When used alone, condoms (prophylactics) have about what failure rate?

DO NOT DISTRIBUTE - 29 -
Human Sexuality 14Mar2009

a) 1%
b) 2%
c) 3%
d) 10%
e) 20%
10) Coitus interruptus (withdrawal method) has a 20% failure rate. Douching, which can
actually propel sperm into the uterus, has what failure rate for contraception?
a) 10%
b) 20%
c) 40%
d) 60%
e) 80%
11) Which of the following fertility awareness techniques has about a 20% failure rate in
the first year and involve testing urine for LH?
a) Calendar method (peak days)
b) Basal body temperature (BBT)
c) Cervical mucous (ovulation) method
d) Ovulation-Prediction kits
12) Years after a vasectomy procedure, a man remarries and wants to have children.
Which of the following complications of the vasovasectomy procedure can occur, leading
to difficulty conceiving?
a) The body can no longer produce sperm
b) The body can produce antibodies toward the sperm
c) The vasculature to the vas deferens has been damaged and thus they are no
longer viable
d) The vas deferens are permanently blocked
e) The sperm no longer have an affinity toward the ova
13) The contraceptive that involves a long-acting, synthetic form of progesterone is
known as:
a) RU486 (Mifepristone)
b) Norplant
c) Depo-provera (DMPA)
d) Nuva ring
e) IUD
14.1) Ancient Egyptians used crocodile dung as a contraceptive.
a) True
b) False
14.2) There is an oral contraceptive that can be taken two days after unprotected
intercourse.
a) True
b) False
14.3) Sterilization operations can be surgically reversed.
a) True
b) False
14.4) Testosterone can be used as a male contraceptive.
a) True

DO NOT DISTRIBUTE - 30 -
Human Sexuality 14Mar2009

b) False
14.5) Abortions were legal in the newly founded United States.
a) True
b) False
14.6) Contraceptives prevent contraception as well as the spread of sexual transmitted
infections (STIs).
a) True
b) False
14.7) The dilation and curette (D & C) is the most widely used abortion method in the
United States.
a) True
b) False
15.1) Which of the following is true of the contraceptive minipill?
a) Contains synthetic progesterone and estrogen
b) Contains synthetic progesterone
c) Contains synthetic estrogen
15.2) Which of the following is a kind of tubal sterilization in which the fallopian tubes
are approached through an incision in the back wall of the vagina.
a) Complete hysterectomy
b) Partial hysterectomy
c) Culpotomy
d) Hysterotomy
e) Laproscopy
f) Minilaparotomy
15.3) The dilation and evacuation (D & E), is a common abortion method in which the
cervix is dilated prior to vacuum aspiration
a) True
b) False
16.1) The best time to talk with your partner about contraception is anytime that allows
your contraceptive to become effective before you engage in coitus.
a) True
b) False
16.2) The majority of Americans feel that abortion should not occur after the first two
trimesters.
a) True
b) False
16.3) Most Americans believe abortion should be permitted but there is great controversy
over partial-birth abortion.
a) True
b) False

13 – Sexuality in Childhood and Adolescence


1) Which of the following is the final indicator for infant sexual response?
a) Sexual curiosity
b) Genital play
c) Masturbation

DO NOT DISTRIBUTE - 31 -
Human Sexuality 14Mar2009

2) When does masturbation typically begin for humans?


a) 3 to 6 months
b) 6 to 12 months
c) 2 years old
d) 4 years old
e) 6 years old
3) Scolding and punishing children who masturbate has no long-term affect on sexual
behavior.
a) True
b) False
4.1) According to Friedrich’s study, what is a common behavior for children ages 2 to 5?
a) Asking about kissing
b) Touching their mother’s breasts
c) Touching their private parts at home
d) Touching the private parts of relatives
e) Touching the breasts of strangers
4.2) According to Friedrich’s study, what is a common behavior for children ages 6 to 9?
a) Asking about kissing
b) Touching their mother’s breasts
c) Touching their private parts at home
d) Touching the private parts of relatives
e) Touching the breasts of strangers
5) Same sex play during childhood, such as touching genitals or oral-genital contact, does
not foreshadow adult sexual orientation.
a) True
b) False
6) The problem with most sex education modules in America is that it mainly focuses on
biological aspects of puberty and reproduction, but rarely deals with abortion,
masturbation, sexual orientation, or sexual pleasure.
a) True
b) False
7) Which of the following is NOT a suggestion by Calderone and Johnson for improving
parent-child communication about sexuality?
a) Be willing to answer questions about sex
b) Use necessary, and even vulgar, language
c) Give advice in the form of information that the child can use to make sound
decisions upon
d) Share information in small doses
e) Encourage the child to talk about sex
8) Why has the age of menarche declined among girls in Western nations?
a) An increase in fast-food diets
b) The use of alcohol at earlier ages
c) A decrease in physical activity
d) Teens are having sex more often
e) Better nutrition and health care
9) In what time frame does pubic hair begin to grow and cover the mons?

DO NOT DISTRIBUTE - 32 -
Human Sexuality 14Mar2009

a) Ages 8 - 11
b) Ages 9 - 15
c) Ages 10 - 16
d) Ages 12 - 19
10) In what time frame does the first ejaculation occur?
a) Ages 9 - 15
b) Ages 11 - 16
c) Ages 11 - 17
d) Ages 14 - 18
11.1) African American males are more likely than Latino American and European
American males to have premarital sexual intercourse.
a) True
b) False
11.2) African American males are more likely than Latino American and European
American males to have engaged in cunnilingus.
a) True
b) False
12) Which of the following is a biological reason for adolescence engaging in sex?
a) To punish their parents
b) As a reward to guys for being loyal
c) Sexual arousal hormones
d) Because of love or desire
e) To show a dominant role
13) Which of the following would NOT decrease the likelihood that female adolescents
will be sexually active?
a) Adolescents that are more career orientated
b) Children in a two-parent household
c) Drug use
d) Family values
e) Religion
14) About what percent of adolescence report sexual experiences with people of their
own sex?
a) 1%
b) 2%
c) 5%
d) 10%
e) 50%
15) The recurrent themes for teen mothers are poverty, joblessness, and a lack of hope for
the future, with half of them quitting school. What percent of American girls between the
ages of 15 and 19 become pregnant each year?
a) 1%
b) 2%
c) 5%
d) 10%
e) 50%

DO NOT DISTRIBUTE - 33 -
Human Sexuality 14Mar2009

16) Which of the following is NOT a factor that contributes to the incidence of teen
pregnancy?
a) Loosening of traditional taboos
b) Lack of religion
c) Impaired family relationships
d) Problems in school
17) Adolescents often report not using contraceptives because they feel that not having
intercourse often enough warrants their use. They also report not wanting to disrupt sex
with the use of contraceptives.
a) True
b) False
18) Most adolescents admit that it would be easier for them to postpone sex and avoid
pregnancy if they could have more open discussions with their parents.
a) True
b) False
19.1) Many boys are born with erections.
a) True
b) False
19.2) Most children learn the facts of life from their parents or from school sex education
programs.
a) True
b) False
19.3) There is evidence that sex education encourages sexual activity.
a) True
b) False
19.4) Erotic dreams in adolescent men always result in nocturnal emission (wet dreams).
a) True
b) False
19.5) Sexual petting is universal among adolescents in the United States.
a) True
b) False
19.6) Nearly 800,000 adolescent girls in the United States become pregnant each year.
a) True
b) False
20.1) The Critical Fat Hypothesis is a view that girls must reach a certain body weigh to:
a) Being sexual contact with the opposite sex
b) Develop secondary sex characteristics
c) Reach menarche
d) Feel comfortable with sexual contact
e) Be sexually attractive to boys
20.2) Personal fable, the belief that one’s feelings and ideas are special and that one is
invulnerable, is most common amongst adolescence.
a) True
b) False
21.1) Communication with your children about tender subjects such as sex is nearly
always a good idea.

DO NOT DISTRIBUTE - 34 -
Human Sexuality 14Mar2009

a) True
b) False
21.2) Most girls are satisfied with their first sexual encounter.
a) True
b) False
22) Because of the lack of education, young African Americans are paying a very steep
price with an increase in STDs including HIV/AIDS.
a) True
b) False
23) Internet access and technology is a double-edged sword for parents today. It allows
parents to learn about adolescent sexual topics but also provides a means for teenagers to
access adult content.
a) True
b) False

14 – Sexuality in Adulthood
1) Which of the following types of marriage is legal in the United States?
a) Monogamy
b) Polygyny
c) Polyandry
d) B & C
e) A, B, & C
2) Which of the following in 2008 does NOT allow full marriage rights to same sex
couples?
a) Netherlands
b) Belgium
c) Canada
d) United States
3) Today, married couples engage in coitus more frequently, with greater variety, and for
longer periods of time.
a) True
b) False
4) What reason do women give the most for participating in extramarital affairs?
a) More frequent sex
b) Better sex
c) Friendship
d) Love
e) Romance
5) As the composition of the U.S. population gets progressively older, it is important to
be aware of sex in the elderly, as it is important to preventing problems that may arise.
a) True
b) False
6) With fine tuning of expectations, elderly couples may find themselves leading some of
the most sexually fulfilling years of their lives.
a) True
b) False

DO NOT DISTRIBUTE - 35 -
Human Sexuality 14Mar2009

7) Which of the following does NOT often occur with aging?


a) Reduced vaginal lubrication
b) Reduced female orgasm intensity
c) More semen production due to prostate
d) Softer erections
e) Longer refractory periods
8) The availability of a sexually interested and supportive partner may be the most
important determinant of continued sexual activity.
a) True
b) False
9) Which of the following is NOT a factor that Nosek identifies as central to sexual
wellness among the disabled?
a) Positive sexual self-concept
b) Knowledge about sexuality
c) Positive, productive relationships
d) Ignoring current barriers to sexuality
e) Maintaining the best possible general and sexual health
10) Which of the following disabilities leads to inability to perform sexually?
a) Cerebral palsy
b) Lose vaginal sensation or lubrication
c) Spinal cord injury males
d) Arthritis
e) None of the above
11) Regardless of physical, sensory, or psychological disability, people just need time to
develop a comfort zone that applies to their sexuality.
a) True
b) False
12.1) Singlehood has become a more common U.S. lifestyle over the past few decades.
a) True
b) False
12.2) Divorced people are more likely than never married people to cohabitate.
a) True
b) False
12.3) In the ancient Hebrew and Greek civilizations, wives were viewed as their
husband’s property.
a) True
b) False
12.4) Men are more romantic, with regard to love at first sight and believing in soul
mates, than women.
a) True
b) False
12.5) Most of today’s sophisticated young people see nothing wrong with an occasional
extramarital fling.
a) True
b) False
12.6) Men are more likely than women to commit acts of domestic violence.

DO NOT DISTRIBUTE - 36 -
Human Sexuality 14Mar2009

a) True
b) False
12.7) Few women can reach orgasm after the age of 70.
a) True
b) False
12.8) People who are paralyzed as a result of spinal cord injuries cannot become sexually
aroused or engage in coitus.
a) True
b) False
13.1) Swinging or “mate-swapping” is also known as:
a) Polygamy
b) Polyandry
c) Polygyny
d) Comarital sex
e) Extramarital sex
13.2) The mating gradient is the tendency for women to “marry-up” and for men to
“marry-down” with regard to social and economic status.
a) True
b) False

15 – Sexual Dysfunctions
1) Which of the following Diagnostic and Statistical Manual of Mental Disorders (DMD)
categories of sexual dysfunction involves dyspareuia?
a) Sexual desire disorders
b) Sexual arousal disorders
c) Orgasm disorders
d) Sexual pain disorders
2.1) Hypogonadism (small testes) or removal or the adrenal gland could result in which
type of disorder?
a) Sexual desire disorder
b) Sexual arousal disorder
c) Orgasm disorder
d) Sexual pain disorder
2.2) If a women can reach orgasm during masturbation or oral sex, they can reach orgasm
through coitus with their partner.
a) True
b) False
2.3) Vaginismus, which involves involuntary contraction of the pelvic muscles that
surround the outer third of the vaginal barrel and is most often caused by a psychological
fear of penetration, is considered which type of disorder?
a) Sexual desire disorder
b) Sexual arousal disorder
c) Orgasm disorder
d) Sexual pain disorder
3) Which of the following organic factors is most associated with erectile dysfunction?
a) Alcohol use

DO NOT DISTRIBUTE - 37 -
Human Sexuality 14Mar2009

b) Heroine use
c) High cholesterol
d) Morphine
e) Cocaine use
4) Which of the following factors would NOT cause sexual dysfunction?
a) Emotional factors
b) Psychosexual trauma
c) Sexual orientation
d) Lack of sexual skill
e) Familial factors
5) What is the goal of sex therapy?
a) To help the couple achieve orgasm
b) To help the couple procreate
c) To introduce the couple to new sexual experiences
d) To incorporate sexual toys
e) To change self-defeating beliefs and attitudes
6) When looking for a sex therapist, it is important to find someone with specialized
training and current certification and licensure.
a) True
b) False
7.1) Alprostadil is used to treat sexual arousal disorder via what application?
a) Applying to the anus
b) Inserting into the penile urethra
c) Ingestion (pill)
d) Injection (IM)
e) Transdermal patch
7.2) How does Apomorphine (Uprima) treat sexual arousal disorders?
a) Increases dopamine, leading to erection
b) Decreases dopamine, leading to erection
c) Increase norepinephrine, leading to vaginal secretions
d) Decreases norepinephrine, leading to vaginal secretions
8) A testosterone patch can enhance sexual arousal in females.
a) True
b) False
9.1) A common method of treating sexual orgasm disorders is a couples-orientated
approach where the woman guides the man in caressing and movements.
a) True
b) False
9.2) What is the role of clomipramine in sexual disorders?
a) Increases the strength of female orgasm
b) Increases the strength of male orgasm
c) Prevents premature female orgasm
d) Prevents premature male orgasm
e) Allows for deeper coital penetration
10) Dyspareuina (painful intercourse) is usually treated by managing the underlying
physical problem, such as a urinary tract infection.

DO NOT DISTRIBUTE - 38 -
Human Sexuality 14Mar2009

a) True
b) False
11) The development of treatments for sexual dysfunction for women has lagged behind
men due to a lack of understanding of the female sexual dysfunctions and it has been
historically a male driven society for drugs of this nature to get attention.
a) True
b) False
12.1) Sexual dysfunctions are rare.
a) True
b) False
12.2) Only men can reach orgasm too early.
a) True
b) False
12.3) The most common cause of painful intercourse in women is vaginal infection.
a) True
b) False
12.4) Sex therapy teaches a man with erectile disorder how to will an erection.
a) True
b) False
12.5) A doctor made a somewhat unusual presentation to a medical convention by
dropping his pants to reveal an erection.
a) True
b) False
12.6) Many sex therapists recommend masturbation as the treatment for women who
have never been able to reach orgasm.
a) True
b) False
12.7) A man can be prevented from ejaculating by squeezing his penis when he feels the
he is about to ejaculate.
a) True
b) False
13.1) A spectator role, in which people observe rather than fully participate in their
sexual encounters, is usually taken because of performance anxiety.
a) True
b) False
13.2) Exercises in which sex partners take turns giving and receiving pleasurable
stimulation in nongenital areas of the body are called:
a) Tantric
b) Tumescent
c) Squeeze
d) Sesate focus
e) Vaginismus
14.1) Dhat Syndrome is found among young Asian males and involves excessive fears
over the loss of seminal fluid during nocturnal emissions.
a) True
b) False

DO NOT DISTRIBUTE - 39 -
Human Sexuality 14Mar2009

14.2) Comparing cultures, in Mangia premarital relations do not occur and the Inis Beag
are taught sexual techniques are a young age.
a) True
b) False

16 – Sexually Transmitted Infections


1) The United States is believed to have the highest rate of sexually transmitted disease
(STIs) in the industrialized world
a) True
b) False
2.1) Which of the following is considered a fungal infection?
a) Vaginosis (BV)
b) Chlamydia
c) Tricomoniasis
d) Syphilis
e) Gonorrhea
2.2) Which of the following is considered a fungal infection?
a) Genital herpes
b) Hepatitis
c) HIV
d) Genital warts
e) Candidiasis
2.3) Viral herpes can be cured with drugs such as acyclovir and famciclovir.
a) True
b) False
3) Which of the following is transmitted exclusively with sexual contact?
a) Scabies
b) Pediculosis (crabs)
c) Genital warts
d) Candidiasis
e) Syphilis
Match the common symptom with the disease:
4.1) Yellow, thick, burning penile discharge a) Scabies
4.2) Hard, round painless chancre b) Crabs
4.3) Painful urination, mostly asymptomatic c) Warts
4.4) Thin, foul smelling discharge d) HIV
4.5) Vulva itching, foul discharge e) Hepatitis
4.6) Foamy, yellow, odorous discharge f) Herpes
4.7) Painful, reddish bumps, prodromal symptoms g) Tricomoniasis
4.8) Asymptomatic to mild flu-like symptoms h) Candidiasis
4.9) Fever, weight loss, rare forms of cancer i) Bacterial vaginosis
4.10) Painless warts, cauliflower shaped j) Chlamydia
4.11) Itching in pubic hair area k) Syphilis
4.12) Intense itching, reddish lines on skin l) Gonorrhea
5.1) Which of the following requires 3 of 4 diagnostic markers and includes Clue cells?
a) Gonorrhea

DO NOT DISTRIBUTE - 40 -
Human Sexuality 14Mar2009

b) Bacterial vaginosis
c) Tricomoniasis
d) HIV
e) Crabs
5.2) How is genital herpes diagnosed?
a) Fluid drawn from the base of a genital sore
b) Microscopic examination
c) Abbott Testpack
d) VDRL blood test
e) Antibodies
5.3) How is viral hepatitis diagnosed?
a) Fluid drawn from the base of a genital sore
b) Microscopic examination
c) Abbott Testpack
d) VDRL blood test
e) Antibodies
Match the disease with the drug of choice:
6.1) Gonorrhea a) Acyclovir
6.2) Syphilis b) Metronidazole
6.3) Chlamydia c) Ceftriaxone
6.4) Bacterial vaginosis d) Penicillin
6.5) Herpes e) Azithromycin
7) Which of the following could most help reduce the risk of spreading STDs?
a) Anal rather than vaginal intercourse
b) Using the withdrawal method
c) Birth control pills
d) Oral-genital stimulation
e) Condoms
8) Gardnerella vaginalis is associated with which of the following?
a) Gonorrhea
b) Tricomoniasis
c) Candidiasis
d) Bacterial vaginosis
e) Syphilis
9) What is the recommended treatment for genital warts?
a) Azithromycin
b) Ceftriaxone
c) Ciprofloxacin
d) Metronidazole
e) Acyclovir
f) Cryotherapy
g) Fluconazole
10) What is a likely result of a gonorrhea infection that is not treated?
a) Excessive ovulation
b) Pancreatitis
c) Pelvic inflammatory disease (PID)

DO NOT DISTRIBUTE - 41 -
Human Sexuality 14Mar2009

d) Damage to renal nephrons


e) Blindness
11) A patient presents with a rash on their palms and foot soles. Blood tests are positive
for Syphilis. What stage is this patient in?
a) First
b) Second
c) Third
d) Early forth
e) Late forth
12) Which of the following has the highest risk for transmitting HIV?
a) Oral sex on a woman
b) Oral sex on a man
c) Needle drug use
d) Vaginal intercourse
e) Lesbian sexual interactions
13) HIV uses gp120 spikes to bind to sites on cells in the immune system, eventually
crippling the immune system. At what point does HIV become AIDS?
a) CD4 < 50 cells per cubic mm
b) CD4 < 100 cells per cubic mm
c) CD4 < 200 cells per cubic mm
d) CD4 < 500 cells per cubic mm
e) CD4 < 1,000 cells per cubic mm
14) HIV is often transmitted through insect and animal bites and can be transmitted in
work environment such as from bumping into someone or at drinking fountains.
a) True
b) False
15) Which of the following psychosocial factors would not underlie risky sexual behavior
among young people?
a) Perceived low risk of infection
b) Absense of religion in the household
c) Negative attitudes toward condom use
d) Feeling of personal invulnerability
16.1) Most women who contract gonorrhea do not develop symptoms.
a) True
b) False
16.2) Christopher Columbus brought more than beads, blankets, and tobacco back to
Europe from the New World: he also brought syphilis.
a) True
b) False
16.3) Gonorrhea and syphilis can be contracted from toilet seats in public rest rooms.
a) True
b) False
16.4) If a syphilitic sore goes away by itself, the infection does not require medical
treatment.
a) True
b) False

DO NOT DISTRIBUTE - 42 -
Human Sexuality 14Mar2009

16.5) Men can develop vaginal infections.


a) True
b) False
16.6) Most people who are infected with HIV remain asymptomatic and appear healthy
for years.
a) True
b) False
16.7) Genital herpes can be transmitted only during flare-ups of infection.
a) True
b) False
17.1) HAART therapy is associated with which of the following?
a) Genital herpes
b) Hepatitis
c) HIV
d) Genital warts
e) Candidiasis
17.2) Donovanosis (granuloma inguinale) is a bacterial disease common in many
underdeveloped regions and has painless genital ulcers which can be mistaken for:
a) Candidiasis
b) Chlamydia
c) Tricomoniasis
d) Syphilis
e) Gonorrhea
17.3) What type of infection is Molluscum contagiosum?
a) Bacterial
b) Viral
c) Ectoparasite
d) Fungi
17.4) Which of the following causes Lymphogranuloma venereum (LGV)?
a) Candidiasis
b) Chlamydia
c) Tricomoniasis
d) Syphilis
e) Gonorrhea

17 – Atypical Sexual Variations


Covered in Behavioral Health Module

18 – Sexual Coercion
1) Although rare, when a women is involved in rape of another person she is often aiding
or abetting men who are attacking a woman.
a) True
b) False
2) Most men who rape other men are heterosexual and sexual motives are often absent.
a) True
b) False

DO NOT DISTRIBUTE - 43 -
Human Sexuality 14Mar2009

3) Which of the following is NOT true of incarcerated rapists?


a) Most rapists are in control of their behavior
b) Most rapists know their behavior is illegal
c) Most rapists are mentally ill
d) Some rapists have long histories of violence
e) For some rapists, violence and sex become intermeshed
4) Which of the following is NOT a suggestion from Powell to provide for individuals
who have been raped?
a) Shower to help feel less humiliated
b) Consider reporting the incident to police
c) Ask a friend or relative to take you to the hospital
d) Seek help in an assertive way
e) Question health professionals about risks and treatments available
5) Child molesters and pedophiles are nearly always male.
a) True
b) False
6) Incest taboo helps reduce genetic defects and thus promotes survival of the species.
a) True
b) False
7) What is the most common form of incest?
a) Mother-son
b) Father-daughter
c) Brother-sister
8) A common scenario for incest is a dominant father who has been rejected sexually by
his wife, and turns to his daughter for emotional and sexual support.
a) True
b) False
9) Which of the following is NOT part of sexual harassment?
a) Deliberate unsolicited comments
b) Repeated unwelcome gestures
c) Unsolicited physical contact
d) Unwelcome contact
e) Lunch with a coworker of the opposite sex
10.1) A woman is raped every 10 minutes.
a) True
b) False
10.2) Rape is ten times greater in the U.S. than Japan.
a) True
b) False
10.3) A majority of rapes are committed by strangers.
a) True
b) False
10.4) When women say no, they sometimes mean yes.
a) True
b) False
10.5) Most rapists are mentally ill.

DO NOT DISTRIBUTE - 44 -
Human Sexuality 14Mar2009

a) True
b) False
10.6) It is unclear whether women should attempt to fight off a rapist.
a) True
b) False
11) A rape that is motivated by the desire to control and dominate the person assaulted is
known as:
a) Acquaintance rape
b) Sadistic rape
c) Anger rape
d) Power rape
e) Statutory rape

19 – Commercial Sex
1) Those working for escort services as well as call girls are usually:
a) Well educated and from lower class families
b) Well educated and from middle class families
c) Well educated and from upper class families
d) Poorly educated and from lower class families
e) Poorly educated and from middle class families
2) Which of the following is NOT a reason why men seek to purchase services from
prostitutes?
a) Sex without negotiation or commitment
b) Sex for eroticism and variety
c) A way to socialize
d) Sex away from home
e) A way to prevent pregnancy
3.1) Prostitution is illegal throughout the entire United States.
a) True
b) False
3.2) Nearly all massage and escort services that advertise in the Yellow Pages are fronts
for prostitution.
a) True
b) False
3.3) It appears that the majority of female prostitutes were sexually abused as children.
a) True
b) False
3.4) Only males are sexually aroused by pornography and other prurient material.
a) True
b) False
3.5) Porn does not make men violent, but can make violent men more violent.
a) True
b) False
3.6) About 25% of all Internet surfing is devoted to the pursuit of porn, adult chat rooms,
and the like.
a) True

DO NOT DISTRIBUTE - 45 -
Human Sexuality 14Mar2009

b) False
4.1) Men who engage in prostitution with male customers are called:
a) Call girls
b) Hustlers
c) Pimps
d) Scores
e) Streetwalkers
4.2) The Whore-Madonna complex is a rigid stereotyping of women as either sinners or
saints.
a) True
b) False

20 – Introduction To Human Sexuality & Adjunctive Material


1) Management of which of the following medical conditions can directly affect sexual
function?
a) Vascular disease
b) Neurological dysfunction
c) Diabetes
d) Arthritis
e) All of the above
2) In the Tanner Stages of development (Sex Maturity Ratings, SMRs), what stage is
denoted by pubic hair spreading to the medial surfaces of thighs?
a) SMR1
b) SMR2
c) SMR3
d) SMR4
e) SMR5
3) In general, SMR classification for boys uses their ____ and for girls uses their ____.
a) Penis; Vagina
b) Penis; Breasts
c) Testicles; Vagina
d) Testicles; Breasts
e) Voice; Self-image
4) Physicians should not assume their patients understand medical terms and thus should
use which of the following?
a) Simple language only
b) Simple language mostly and explicit language as necessary
c) Simple language and explicitly language freely
d) Simply language as necessary and explicit language mostly
e) Explicit language only
5) Which of the following should NOT be true when taking a sexual patient history?
a) Choose a comfortable setting
b) Care must be taken when choosing words to use
c) Assure the patient of confidentiality
d) Be judgmental
e) Have a respectful attitude

DO NOT DISTRIBUTE - 46 -
Human Sexuality 14Mar2009

6) According to “The Proactive Sexual Health History,” what is the first screening
question to ask during a sexual history?
a) Have you ever been sexually involved with anyone?
b) What sexual concerns do you have?
c) Have you been sexually involved with anyone in the past six months?
d) With men, women, or both?
e) If you were sexually active, do you imagine it would be with men, women, or
both?
7) The estimated prevalence of sexual dysfunction (2002) in the general population is as
high as 52 percent in men and 63 percent in women. Sexual concerns have been reported
in 75% of couples seeking marital therapy and about what percentage of women seeking
routine gynecologic care?
a) 0%
b) 25%
c) 50%
d) 75%
e) 100%
8) Which of the following is NOT a preventive sexual health question?
a) Do you use needles to inject street drugs?
b) How do you protect yourself from HIV and other STDs?
c) Have you ever been tested for HIV? Would you like to be?
d) Do you use anything to prevent pregnancy? Are you satisfied with that
method?
e) Have you ever been immunized against hepatitis>? Would you like to be?
9) In the 2003 U.S. Supreme Court decision Lawrence vs. Texas, what type of sodomy
laws were invalidated in the fifty states?
a) Sex with dead bodies
b) Oral sex
c) Anal sex
d) Beastiality
e) Homosexual sex
10) The Center for Disease Control (CDC) shows that over what percentage of men and
women engage in heterosexual anal stimulation?
a) 5%
b) 10%
c) 15%
d) 20%
e) 30%
11) The Food and Drug Administration (FDA) approved Aneros device is meant to
stimulate which of the following?
a) Female G-spot (Grafenberg spot)
b) Penis
c) Testicles
d) Prostate
e) Clitoris
12) What are intra vaginal balls (Ben-Wa balls) used for?

DO NOT DISTRIBUTE - 47 -
Human Sexuality 14Mar2009

a) Sexual stimulation for the female


b) Sexual stimulation for the male partner
c) Strengthening the pubococcygeus muscle
d) Preventing yeast infections
e) A & C
13) Penis rings are placed behind the testicles and over the dorsal shaft to prolong
erection. Damage can occur if these rings are used for longer than:
a) Thirty-seconds
b) One-minute
c) Five-minutes
d) Thirty-minutes
e) Twenty-four hours
14) What sexual position is recommended for pregnant or obese patients?
a) Rear-entry (“doggy style”)
b) Female superior (“cowgirl”)
c) Male superior (“missionary”)
e) Sitting (“lotus”)
15) What sexual position is recommended for a male patient who recently had a heart
attack?
a) Rear-entry (“doggy style”)
b) Female superior (“cowgirl”)
c) Male superior (“missionary”)
d) Lateral recumbent
e) Sitting (“lotus”)
16) Which of the following positions is the worst for women with back complaints?
a) Rear-entry (“doggy style”)
b) Female superior (“cowgirl”)
c) Male superior (“missionary”)
d) Lateral recumbent
e) Sitting (“lotus”)
17) Pre-menopausal women should perform their breast self-exam (BSE):
a) 5-10 days prior to menstruation
b) Just prior to menstruation
c) During menstruation
d) Just after menstruation
e) 5-10 days after menstruation
18) Which of the following symptoms in the “ACHES” pain mnemonic can signify
possible gallbladder disease
a) Abdominal pain
b) Chest pain
c) Headache
d) Eye problems
e) Severe leg pain
19) The CDC advices NOT to use which of the following associated with condoms?
a) Ribbed condoms
b) Water-based lubricant

DO NOT DISTRIBUTE - 48 -
Human Sexuality 14Mar2009

c) Nonoxynol-9
d) Latex
e) Rubber
20) Increased vaginal lubrication as well as an engorged vagina are most likely seeing
during which trimester of pregnancy?
a) First
b) Second
c) Third
21) The CDC estimates that ____ of HIV infected patients are over the age of 50?
a) 0.5%
b) 1%
c) 2%
d) 5%
e) 10%
22) A 1979 study showed ____ lesbians came to terms with their sexuality after age 50.
a) All
b) 1 in 2
c) 1 in 3
d) 1 in 4
e) No
23) Which of the following is NOT a normal age-related sexual change?
a) Longer interval to ejaculate
b) Longer post orgasm refractory period for men
c) Delayed and more firm erection
d) Reduced vaginal expansion
e) Reduced vaginal lubrication
24) If a woman has bacterial vaginosis (BV) ____ time(s) with the same sex partner, than
the partner should be treated as well.
a) 1
b) 2
c) 3
d) 4
e) 5
25) A patient present with vaginal itching and burning. An elevated pH (>4.5) is found on
the anterior vaginal fornix, a positive KOH “whiff” test elicits a fish odor, and clue cells
are found under microscopy. Which of the following drugs should be given to this
patient?
a) Azithromycin
b) Ceftriaxone
c) Ciprofloxacin
d) Metronidazole
e) Acyclovir
f) Cryotherapy
g) Fluconazole

DO NOT DISTRIBUTE - 49 -
Human Sexuality 14Mar2009

26) A patient presents with painful “kissing” ulcerations on their penis. Laboratory
results come back positive for Haemophilus ducreyi. What drug should the patient be
given for their chancroid?
a) Azithromycin
b) Ceftriaxone
c) Ciprofloxacin
d) Metronidazole
e) A or B
f) B or C
g) C or D
27) A patient presents with clear discharge and no other sexual symptoms. A DNA Probe
tests positive for Chlamydia trachomatis. What drug should this patient be given?
a) Azithromycin
b) Ceftriaxone
c) Ciprofloxacin
d) Metronidazole
e) Acyclovir
f) Cryotherapy
g) Fluconazole
28) If a patient is taking Metronidazole, what should they avoid doing?
a) Exercising vigorously
b) Eating fatty foods
c) Eating shellfish
d) Drinking alcohol
e) Smoking cigarettes
29) A patient presents with dysuria and penile discharge. Chocolate agar in the lab
reveals Neisseria gonorrhoeae. What drug should the patient be given?
a) Azithromycin
b) Ceftriaxone
c) Ciprofloxacin
d) Metronidazole
e) Acyclovir
f) Cryotherapy
g) Fluconazole
30) Patients who present with gonorrhea should also be treated for:
a) Herpes
b) HIV
c) Chlamydia
d) Bacterial vaginosis
e) Syphilis
31) A patient presents with painful genital blisters that come and go with time. What
should they be given?
a) Azithromycin
b) Ceftriaxone
c) Ciprofloxacin
d) Metronidazole

DO NOT DISTRIBUTE - 50 -
Human Sexuality 14Mar2009

e) Acyclovir
f) Cryotherapy
g) Fluconazole
32) A patient presents with cauliflower-like genital warts and is diagnosed with human
papillomaviris (HPV). What is the recommended treatment?
a) Azithromycin
b) Ceftriaxone
c) Ciprofloxacin
d) Metronidazole
e) Acyclovir
f) Cryotherapy
g) Fluconazole
33) The drug pedophyllin should NOT be used with:
a) Penile warts
b) Scrotal warts
c) Labial warts
d) Perineal warts
e) Anal warts
34) Non-specific cervicitis (NSC) and mucopurulent cervicitis (MPC) require at least
____ white blood cells (WBCs) per high-power film (hpf) for diagnosing in females.
a) 2
b) 3
c) 5
d) 8
e) 10
35) Non-gonococcal urethritis (NGU) and non-specific urethritis (NSU) require at least
____ WBCs per hpf Gram stain for diagnosis in males.
a) 2
b) 3
c) 5
d) 8
e) 10
36) What drug is given for NSC and MPC?
a) Azithromycin
b) Ceftriaxone
c) Ciprofloxacin
d) Metronidazole
e) Penicillin
f) Fluconazole
37) Which of the following requires a broad range of suspicion for diagnosis and is the
leading cause of infertility and ectopic pregnancy?
a) Non-specific cervicitis (NSC)
b) Mucopurulent cervicitis (MPC)
c) Pelvic inflammatory disease (PID)
d) Trichomoniasis
e) Candidiasis

DO NOT DISTRIBUTE - 51 -
Human Sexuality 14Mar2009

38) A patient presents with a rash on their palms and soles of their feet. History reveals
they had a genital chancre, but did not seek help because it was painless. What drug
should this patient be given as a single dose?
a) Azithromycin
b) Ceftriaxone
c) Ciprofloxacin
d) Metronidazole
e) Penicillin
f) Fluconazole
39) For late latent (greater than one year) or tertiary (non-neuro) Syphalis, how many
dosages of the recommended drug are given over consecutive weeks?
a) 1
b) 2
c) 3
d) 4
e) 5
40) A patient presents with green, sometimes gray, frothy discharge. Wet mount
visualization shows Trichomonas vaginalis. What drug should they be given?
a) Azithromycin
b) Ceftriaxone
c) Ciprofloxacin
d) Metronidazole
e) Penicillin
f) Fluconazole
41) Which of the following presents as a glans penis “rash,” may be associated with
uncircumcised males, and requires Mycolog (Nystatin and Triamcinolone Acetonide
Cream) treatment?
a) PID
b) Trichomoniasis
c) Balanitis
d) Candidiasis
e) Hepatitis
42) A female patient presents with a white creamy “cottage cheese” discharge. A KOH
prep is used to diagnose candidiasis (yeast infection). The patient says they tried a 3-day
vaginal cream last week. Which of the following drugs should be given as a single dose?
a) Azithromycin
b) Ceftriaxone
c) Ciprofloxacin
d) Metronidazole
e) Penicillin
f) Fluconazole
43) Which of the following is considered a low risk factor for Hepatitis C infection?
a) Multiple sexual partners
b) Injecting drug users
c) Hemodialysis patients
d) Recipients of clotting factors prior to 1987

DO NOT DISTRIBUTE - 52 -
Human Sexuality 14Mar2009

e) Recipients of blood transfusions prior to 1992


44) Which of the following types of drugs are NOT associated with female sexual
dysfunction (FSD)?
a) Antipsychotics (e.g. SSRIs)
b) Antihypertensives
c) Anticholinergics
d) Hormonal preparations (e.g. Danazol)
e) IM testosterone
45) Which of the following types of drugs is associated with erectile dysfunction (ED)?
a) Cardiac
b) Antihypertensives
c) Antidepressants
d) Sedatives
e) Antiandrogens
f) All of the above
46) Which of the following drugs would be contraindicated with drugs such as Sildenafil
(Viagra), Vardenasil (Levitra), and Tadalafil (Cialis)?
a) Beta blockers
b) Digoxin
c) Lithium
d) Nitroglycerine
e) Dilantin
47) Which of the following is NOT true of body piercing recommendations by the
American Academy of Dermatology?
a) Universal precautions per the Occupational Safety and Health Administration
(OSHA) must be adhered to
b) Parental consent and supervision is required under the age of 18
c) Those getting pieced cannot have a blood alcohol content (BAC) > 0.10
d) Wound care instructions must be given and explained
e) Pre-piercing consultation must describe risks and complications
48) Sex in hot tubs is associated with which of the following?
a) Bacterial vaginosis
b) Pelvic inflammatory disease
c) Trichomoniasis
d) Balanitis
e) Candidiasis
49) Male masturbation with a vacuum cleaner has been shown to:
a) Be a safe form of masturbation
b) Cause local trauma but no lasting effects
c) Cause the penis to lengthen and increase in girth over time
d) Cause erectile dysfunction
e) Cause severe injury including decapitation and loss of the glans penis
50) Gerbilling, also known as gerbil stuffing or gerbil shooting, refers to the insertion of
small animals into a man's rectum to stimulate the prostate as in anal sex. Contrary to
popular belief, gerbilling is unknown as an actual sexual practice, and despite apparently

DO NOT DISTRIBUTE - 53 -
Human Sexuality 14Mar2009

widespread public belief and persistent rumors, especially in the 1980s, no verified
medical evidence of gerbilling exists.
a) True
b) False
51) Hypoxic orgasm, which refers or masturbation activity with anoxia for a heightened
climax, as well as autoerotic asphyxia have been associated with unintentional brain
damage and death.
a) True
b) False
52) Bicycle riding has been associated with erectile dysfunction (ED).
a) True
b) False
53) Amyl nitrite (snappers, poppers) is inhaled by some to intensify sexual activities and
orgasm. The drug works as a potent:
a) Bronchodilator
b) Bronchocontrictor
c) Vasodilator
d) Vasoconstrictor
e) Muscle relaxant
54) What does “on the down low” mean regarding male prisoners?
a) They have performed oral sex on another prisoner
b) They have performed oral sex on a prison guard
c) They have had anal or oral sex and admit they are homosexual
d) They have had anal or oral sex and deny they are homosexual
e) They completely abstained from sexual activity while in prison
55) Men who are “on the down low” may not relate to preventative care messages created
for men who identify themselves as homosexual.
a) True
b) False
56) Which of the following sexual side effects is associated with men who take
Metoclopramide or Cimetidine?
a) Testicular swelling
b) Penile shrinking
c) Breast enlargement
d) Increased ejaculation
e) Difficulty achieving orgasm
57) Which of the following is the most likely sexual dysfunction in women with diabetes
mellitus (DM)?
a) Impaired vaginal lubrication
b) Vulval tumescence (swollen)
c) Increased sexual desire
d) Difficulty achieving orgasm
e) Increased fertility
58) Which of the following occurs in 35-59% of men with diabetes mellitus (DM)?
a) Priapism
b) Impotence

DO NOT DISTRIBUTE - 54 -
Human Sexuality 14Mar2009

c) Premature ejaculation
d) Dry run orgasms
e) Increase sex drive
59) Diabetics with erectile dysfunction are more likely to develop which of the
following?
a) Retinopathy
b) Neuropathy
c) Leukopathy
d) Myopathy
e) A & B
60) Which of the following is usually recommended for patients who have received a
coronary artery bypass graft (CABG)?
a) Return to normal sexual activity
b) Increase sexual activity to strengthen the heart
c) Calm, non-athletic sexual activity
d) No sexual activity
61) Onset of erectile dysfunction in a man over 35 years old may be a warning sign of:
a) Neurological disorders
b) Liver dysfunction
c) Arterial disease
d) Spinal disease
e) Diabetes
62) Which of the following is usually recommended for patients who have had a
myocardial infarction with negative catheterization a week ago or four weeks after no
symptoms or EKG changes are noted?
a) Return to normal sexual activity
b) Increase sexual activity to strengthen the heart
c) Calm, non-athletic sexual activity
d) No sexual activity
63) Increased heart rate (HR) and blood pressure (BP) at maximum levels with orgasm
were seen to be less than with modest activity of a non-sexual nature.
a) True
b) False
64) Which of the following is the most likely barrier to sexual activity for obese and
anorexic patients?
a) Hypertension
b) Lethargy
c) Diabetes
d) GI complaints
e) Psychological
65) Performing in a female superior sexual position may have no apparent advantage for
a man with hypertension.
a) True
b) False
66) Patients with rheumatic diseases (osteoarthritis, rheumatoid arthritis) may have which
of the following sexual symptoms?

DO NOT DISTRIBUTE - 55 -
Human Sexuality 14Mar2009

a) Excessive joint movement


b) Increased vaginal moisture
c) Fatigue
d) Priapism
e) Increased libido
67) Post-stroke mechanical problems such as weakness or spasticity may be overcome
with modified sexual techniques.
a) True
b) False
68) Sexual activity has been shown not to provoke seizure episodes in epileptic patients.
a) True
b) False
69) Patients with Alzheimer disease, especially females, are typically regarded as
“hypersexual.”
a) True
b) False
70) One in two men with Alzheimer disease will have erectile dysfunction.
a) True
b) False
71) Gay men may have an increased risk for Giardia lamblia, Entamoeba histolytica,
Shigella species, Campylobacter species, and Hepatitis A due to which of the following?
a) Oral-anal sexual contact
b) Anal-receptive; oral genital contact
c) Smoking; decreased frequency of screening
d) Increased HPV and HCV from anal-receptive
e) Subculture attitudes and pressure toward beauty
72) Lesbians may have an increased risk for certain types of cancer. Which of the
following risk factors is the least likely to be modified for these women to help prevent
cancer?
a) Smoking
b) Increased alcohol use
c) Fewer screening exams
d) Higher Body Mass Index (BMI)
e) Nulliparity
73) Lesbians may have a decreased risk for STDs as well as which of the following types
of cancer?
a) Breast cancer
b) Ovarian cancer
c) Cervical cancer
d) Endometrial cancer
e) Colon cancer
74) Gay men and women are at a higher risk for stroke, coronary artery disease (CAD),
and lung cancer due to which of the following risk factors?
a) Promiscuity
b) Identity crisis and psychological issues
c) Increased stress

DO NOT DISTRIBUTE - 56 -
Human Sexuality 14Mar2009

d) Increase smoking
e) Fewer screening exams
75) The actual or perceived risk and associated fear of entering a homophobic and
potentially abusive medical system outweigh the perceived risk of illness.
a) True
b) False
76) The phrase “I’m with Dorothy” was used during WWII for military men meaning
they were:
a) Hiding from battle
b) Against gay males
c) Against lesbians
d) Under attack
e) Homosexual
77) A female that is used by a gay man to hide behind in social events to avoid suspicion
is termed a:
a) Butch
b) Dyke
c) Frottage
d) Beard
e) Fluffer
78) Terms used to indicate discomfort in the testicles when ejaculation is delayed (sexual
frustration) include blue balls and:
a) Hard on
b) Freeze frame
c) Frigidity
d) Tricking
e) Impotence
79) Butch is a term used to denote:
a) A gay male who is masculine
b) A gay male who is feminine
c) A lesbian who is masculine
d) A lesbian who is feminine
e) A straight male who is very strong
80) Cornholing is a term used to denote:
a) Oral sex
b) Vaginal sex
c) Anal sex
d) Ejaculation
81) The derogatory term “fag hag” refers to:
a) A women who befriends and socializes with gay men
b) A women who befriends and socializes with lesbians
c) A man who befriends and socializes with gay men
d) A man who befriends and socializes with lesbians
82) The terms felching, shrimping, snowballing, and rainbow kiss have which of the
following in common?
a) They involve oral sex

DO NOT DISTRIBUTE - 57 -
Human Sexuality 14Mar2009

b) They involve anal sex


c) They involve vaginal sex
d) They involve semen
83) A hired member of a pornographic video crew whose job is to arouse the male
participants prior to filming is termed a:
a) Butch
b) Dyke
c) Frottage
d) Beard
e) Fluffer
84) Non-consensual rubbing against another person to achieve sexual arousal is called:
a) Felching
b) Frottage
c) Sodomy
d) Rimming
e) Onanism
85) The gay liberation movement started at a small Greenwich Village bar, The
Stonewall Inn, the day after Judy Garland’s funeral in what year?
a) 1828
b) 1917
c) 1939
d) 1969
e) 2008
86) Klismaphilia is the act of sexual play involving:
a) Feces
b) Plants
c) Enemas
d) Animals
e) Dead bodies
87) Necrophilia is the act of sex involving:
a) Feces
b) Plants
c) Enemas
d) Animals
e) Dead bodies
88) The biblical term for masturbation or “spilling the seed on the ground” is:
a) Felching
b) Frottage
c) Sodomy
d) Rimming
e) Onanism
89) The term queen is used to denote:
a) A gay male who is masculine
b) A gay male who is feminine
c) A lesbian who is masculine
d) A lesbian who is feminine

DO NOT DISTRIBUTE - 58 -
Human Sexuality 14Mar2009

e) A straight male who is very strong


90) Sexual activity involving the tongue on the anus is termed:
a) Felching
b) Frottage
c) Sodomy
d) Rimming
e) Onanism
91) Scat (coprophilia) is sexual activity involving:
a) Feces
b) Plants
c) Enemas
d) Urine
e) Dead bodies
92) Different colored bandanas can be placed in a specific back pant pocket to indicate
sexual activity preference. Colored wrist-bands can also be used to indicate sexual
activity or the accomplishment of sexual acts.
a) True
b) False
93) Although there is little significance today, the phrase “left is right and right is wrong”
referred to homosexual preference denoted by:
a) Handkerchief pocket
b) Ear piercing
c) Wrist to wear a watch on
d) Shoe laces
e) Which foot to tap in the bathroom
94) Size queen is a term used to denote:
a) A gay male who chooses a partner based on how strong they are
b) A gay male who chooses a partner based on their penis size
c) A female who chooses a partner based on how strong they are
d) A female who chooses a partner based on their penis size
e) A lesbian who chooses a partner based on how strong she is
95) Transvestite refers to which of the following?
a) Cross-dressing for fun
b) Men who wear women’s clothes
c) Females who dress “in drag”
d) Drag queens
e) All of the above
96) Tricking is a term that denotes:
a) Dressing in drag
b) Having anal sex
c) Having oral sex
d) Selling oneself for sex
e) Lying about one’s sexual preference
97) The terms “water sport” and “golden shower” refer to sexual activity involving:
a) Feces
b) Plants

DO NOT DISTRIBUTE - 59 -
Human Sexuality 14Mar2009

c) Enemas
d) Urine
e) Dead bodies
98) Studies show that individual counseling by physicians is the single most powerful
factor in persuading patients to change unsafe health behaviors.
a) True
b) False
99) Cancer of the testicles is the number one cancer in teenage boys and young men.
a) True
b) False
100) When putting on a condom, it is important to have the tip of the condom tight
against the glans penis.
a) True
b) False
101) The mons veneris is also known as the mons pubis.
a) True
b) False
102) In general, if a woman is nulliparous (has not had children) only the labia majora of
the vagina will be visible upon inspection in anatomical position.
a) True
b) False
103) The labia minora form a fourchette at what anatomical location?
a) Anterior
b) Posterior
c) Lateral
104) Which of the following is the strongest pelvic floor muscle and can cause
vaginismus if it spasms?
a) Iliococcygeus
b) Pubococcygeus
c) Levator ani
d) Puborectalis
105) Honeymoon cystitis is a type of bladder infection associated with:
a) Initial (virgin) sex
b) Frequent vaginal sex
c) Anal sex
d) Cranberry juice
e) Spermicidal lubricated condom
106) Which of the following uterine positions is the most common?
a) Anteverted
b) Midposition
c) Retroverted
107) Cryptochidism refers to a testicle that is:
a) Atrophied
b) Enlarged
c) Bleeding
d) Necrosing

DO NOT DISTRIBUTE - 60 -
Human Sexuality 14Mar2009

e) Undescended
108) Which of the following is just beyond the urethral bulb and contributes about 30%
of the accessory fluid to the seminal ejaculate?
a) Prostate
b) Bulbourethral gland
c) Seminal vesicle
d) Epididymis
e) Seminiferous tubules
109) Which of the following produces secretions during sexual arousal (pre-ejaculate,
pre-come, or “pre-cum”)?
a) Prostate
b) Bulbourethral gland
c) Seminal vesicle
d) Epididymis
e) Seminiferous tubules
110) Kegel exercises are designed to strengthen which of the following muscles?
a) Iliococcygeus
b) Pubococcygeus
c) Levator ani
d) Puborectalis
111) Which of the following is NOT a phase of sexual response?
a) Excitement phase
b) Plateau phase
c) Orgasmic phase
d) Ejaculation phase
e) Resolution phase
112) If penile entry is attempted in a sexually unaroused female, the flaccid labia minora
may be carried into the vaginal opening causing discomfort and possibly trauma.
a) True
b) False
113) Pain in late arousal for a female could indicate which of the following?
a) HIV
b) Clitoris trauma
c) Bartholin retention cyst
d) Menopause
e) Perineal tear
114) Which of the following arteries supplies the erectile tissues as well as the helicine
arteries of the penile corporal bodies?
a) Internal pudendal artery
b) Perineal artery
c) Cavernosal artery
d) Bulbourethral artery
e) Dorsal artery of the penis
115) A drug that inhibits PDE 5, which degenerates cGMP, thus increasing cGMP and
promoting smooth muscle relation would cause which of the following?
a) Sexual dysfunction

DO NOT DISTRIBUTE - 61 -
Human Sexuality 14Mar2009

b) Flaccidity
c) Erection

21 – Human Immunodeficiency Virus (HIV)


1) Within the M (major) group of HIV-1, there are 10 subtypes including clades B and C.
Which of the following is NOT included in clade B?
a) Americas
b) Japan
c) Australia
d) South Africa
e) Caribbean
2) HAART (Highly Active Antiretroviral Therapy) usually includes which of the
following?
a) Nucleoside reverse transcriptase inhibitor (NRTI), non-nucleoside reverse
transcriptase inhibitor (NNRTI), and transcription inhibitor (TI)
b) Nucleoside reverse transcriptase inhibitor (NRTI), transcription inhibitor (TI),
and protease inhibitor (PI)
c) Nucleoside reverse transcriptase inhibitor (NRTI), non-nucleoside reverse
transcriptase inhibitor (NNRTI), and protease inhibitor (PI)
d) Non-nucleoside reverse transcriptase inhibitor (NNRTI), transcription inhibitor
(TI), and protease inhibitor (PI)
3) Which of the following has the highest risk of HIV transmission?
a) Blood transmission
b) Blood product transmission
c) Sexual transmission
d) Vertical transmission (maternal/fetal)
4) Which of the following drugs has been shown to decrease transmission rate to 8.3% if
taken after the 14th week of gestation?
a) AZT, Zidovudine (Retrovir)
b) 3TC, Lamivudine (Epivir)
c) ddC, Zalcitabine (Havid)
d) ddI, Didanosine (Videx)
e) Delaverdine (Rescriptase)
5) Starting in 2007, the Center for Disease Control (CDC) recommends:
a) Asking high risk patients about HIV
b) Asking all patients about HIV
c) Testing high risk patients for HIV
d) Testing all patients for HIV
e) Not allowing patients to “opt out” of testing
6) In Pennsylvania (PA), All HIV positive antibody tests and positive viral loads and low
CD4 counts are reportable to the PA Health Department.
a) True
b) False
7) Pennsylvania Act 148 mandates all of the following for HIV testing EXCEPT:
a) Consent
b) Pretest counseling

DO NOT DISTRIBUTE - 62 -
Human Sexuality 14Mar2009

c) Confirmatory test
d) Notification of sexual partners
e) Posttest counseling
8) An enzyme-linked immunosorbent assay (ELISA) test is used as preliminary HIV
screening with a Western Blot being used for confirmation.
a) True
b) False
9) The HIV window period between initial infection and onset of detectable antibodies is
typically how long?
a) Less than one month
b) One to three months
c) Three to six months
d) Six months to a year
e) At least a year
10) The OraQuick Advance by OraSure detects antibodies to HIV-1 and HIV-2 in about
20 minutes and can use all of the following types of fluid EXCEPT:
a) Oral fluid
b) Semen
c) Whole blood
d) Blood plasma
11) Patients infected with HIV can receive live vaccinations.
a) True
b) False
12) Testing positive for HIV requires ELISA and Western Blot, or a positive viral load
with acute retroviral infection symptoms in primary HIV infection.
a) True
b) False
13) With a positive HIV antibody serology, acquired immune deficiency syndrome
(AIDS) is diagnosed with an opportunistic infection (OI) or a CD4 count less than:
a) 10,000 cells/mm^3
b) 1,000 cells/mm^3
c) 200 cells/mm^3
d) 50 cells/mm^3
e) 1 cell/mm^3
14) Which of the following is the most common symptom associated with Acute
Retroviral Syndrome (PHI)?
a) Fever
b) Lymphadenopathy
c) Pharyngitis
d) Rash
e) Myalgia
15) HIV treatment with HAART should not begin unless a patient is symptomatic with an
opportunistic infection (OI) or with PHI.
a) True
b) False
Match the CD4 cell/mm^3 count with the prophylaxis treatment:

DO NOT DISTRIBUTE - 63 -
Human Sexuality 14Mar2009

16) Toxoplasma and thrush a) 200 CD4 cells/mm^3


17) Pneumocyctis Pneumonia (PCP) b) 100 CD4 cells/mm^3
18) Disseminated Mycobacterium Avium Complex (MAC) c) 50 CD4 cells/mm^3
19) It is important to know the lowest CD4 level prior to HIV treatment, as when anti-
retroviral therapy is stopped the body will easily return to that CD4 level.
a) True
b) False
20) “Viral blips” or transient low level rises have been shown to be significant in
determining AIDS progression.
a) True
b) False
21) HIV viral eradication is possible with today’s drugs.
a) True
b) False
22) Viral reservoirs are the most common in which of the following locations?
a) CNS
b) Genital tract
c) GI associated lymphoid tissue (GALT)
d) Mucosa associated lymphoid tissue (MALT)
e) Spleen
23) Results of different tests, such as bDNA and PCR, are normalized so they are
equivalent for diagnosis.
a) True
b) False
24) What percentage of newly infected individuals have resistance to at least one drug
prior to initiating treatment for HIV?
a) 0.5%
b) 1%
c) 2%
d) 5%
e) 15%
25) A trofile (tropism test) uses M-topic R5 virus to show ____ infection and T-tropic X4
virus to show _____ infection.
a) Early; Terminal
b) Early; Early
c) Early; Late
d) Late; Early
e) Late; Terminal
26) Resistance testing requires samples of >1000 viral load copies and measures majority
variants (20% or more of the viral population). If a reported mutation came back as
M184V, how would this be interpreted?
a) The major amino acid was moved 184 positions
b) At amino acid 184, the normal type “V” has become type “M”
c) At amino acid 184, the normal type “M” has become type “V”
d) Normal type “V” amino acid has moved 184 positions to “M”
e) Normal type “M” amino acid has moved 184 positions to “V”

DO NOT DISTRIBUTE - 64 -
Human Sexuality 14Mar2009

27) Which of the following HIV drug treatment sites affects cellular exiting of HIV?
a) Chemokine receptors
b) Reverse transcriptase
c) Integrase
d) Protease
e) Maturation
28) Which of the following is a side effect of antiretroviral drugs?
a) Wasting syndrome
b) Anemia
c) Dementia
d) Fat redistribution
e) Opportunistic disease
29) HIV treatment should be offered when CD4 counts are:
a) < 50 cells/mm^3
b) < 200 cells/mm^3
c) 200-350 cells/mm^3
d) > 500 cells/mm^3
e) 500-100,000 cells/mm^3
30) Adipose tissue accumulation seen with HAART therapy is mostly distributed around:
a) The brain
b) Viscera
c) The limbs
d) The abdomen
e) Breasts
31) A “Buffalo Hump” or “pseudo-Cushing Syndrome” seen with HAART therapy is
associated with fat accumulating around:
a) The brain
b) Viscera
c) The limbs
d) The abdomen
e) Breasts
32) Which of the following risk factors for HIV positive patients is seen significantly
more in men then women?
a) Insulin resistance
b) Cardiovascular disease
c) Decreased bone mineral deposits
d) Avascular necrosis
e) Anal cancer
33) Which of the following is the treatment of choice for Pneumocystis jiroveci (carinii)
as well as Toxoplasma gondii?
a) Sulfacetamide
b) Trimethoprim
c) Co-trimazole (TMP-SMX)
d) Phenobarbital
e) Clarithromycin (Biaxin)
34) Which of the following is a drug used for Mycobacterium avium complex?

DO NOT DISTRIBUTE - 65 -
Human Sexuality 14Mar2009

a) Sulfacetamide
b) Trimethoprim
c) Co-trimazole (TMP-SMX)
d) Phenobarbital
e) Clarithromycin (Biaxin)
35) vircoTYPE HIV-1 by Vircolab is an HIV-1 analysis system that combines which of
the following?
a) Genotype information
b) Phenotype information
c) Clinical information
d) All of the above
36) A patient test postive for HIV via a serology test. Their CD4 count is at 10,000
copies. Using the CD4 percentage calculation, this patient would be diagnosed with
AIDS if their CD4 count dropped below:
a) 50 copies
b) 140 copies
c) 200 copies
d) 1,400 copies
e) 2,000 copies
37) Which of the following was NOT included in the first successful HAART treatment,
Merck 035 study?
a) AZT
b) ZDV
c) 3TC
d) Indinavir
38) Which of the following PDE-5 inhibitors has the longest half life?
a) Sildenafil (Viagra)
b) Vardenasil (Levitra)
c) Vardenasil (Lexiva)
d) Vardenasil (Lexepro)
e) Tadalafil (Cialis)

22 – Rape Crisis & Sexual Abuse


1) When children are sexually abused, they know their abuser ____ of the time.
a) 15%
b) 25%
c) 50%
d) 75%
e) 85%
2) Preferential abusers (e.g. pedophiles) are the more common type of child sexual
abusers.
a) True
b) False, situational abusers are more common
3) How many women will be sexually assaulted in their lifetime?
a) 1 in 5
b) 1 in 4

DO NOT DISTRIBUTE - 66 -
Human Sexuality 14Mar2009

c) 1 in 3
d) 1 in 2
e) Nearly all
4) How many men will be sexually assaulted in their lifetime?
a) 1 in 8
b) 1 in 6
c) 1 in 4
d) 1 in 2
e) Nearly all
5) Sexual assault is the most under-reported crime and is reported about ____ of the time.
a) 20%
b) 40%
c) 60%
d) 80%
6) Which of the following is NOT a reason why the highest risk period for sexual abuse
is between ages 16 and 24?
a) Alcohol use
b) Getting driver’s license
c) Starting to date
d) More freedom from parents
e) Unsure of what they want sexually
7) What is the age of consent for statutory sexual assault in the state of Pennsylvania?
a) 13
b) 14
c) 15
d) 16
e) 17
f) 18
8) Regarding meaningful consent, a person who is drunk or high on any other substances
cannot consent to sex.
a) True
b) False, the person must be legally intoxicated per state law
9) Which of the following is the most common date rape drug and is seen in over 70% of
rapes on college campuses?
a) Paracetamol (acetaminophen) and diphenhydramine (Tylenol PM)
b) Gamma-Hydroxybutyric acid (GHB)
c) Flunitrazepam (Rohypnol)
d) Ketamine
e) Midazolam
f) Alcohol
10) Which of the following is NOT true of most date rape drugs?
a) Translucent
b) Odorless
c) Non-polar
d) Tasteless
e) Have amnesia-like effects

DO NOT DISTRIBUTE - 67 -
Human Sexuality 14Mar2009

11) Which of the following reduces the gag-reflex and thus can increase the risk of
aspiration and/or choking?
a) Paracetamol (acetaminophen) and diphenhydramine (Tylenol PM)
b) Gamma-Hydroxybutyric acid (GHB)
c) Flunitrazepam (Rohypnol)
d) Ketamine
e) Midazolam
f) Alcohol
12) Which drug is considered ten times stronger than Valium, has associated memory
loss for 8-12 hours, and can last for 2-3 days?
a) Midazolam
b) Paracetamol (acetaminophen) and diphenhydramine (Tylenol PM)
c) Gamma-Hydroxybutyric acid (GHB)
d) Flunitrazepam (Rohypnol)
e) Ketamine
13) Which of the following drugs is associated with hallucinations and flashbacks?
a) Midazolam
b) Paracetamol (acetaminophen) and diphenhydramine (Tylenol PM)
c) Gamma-Hydroxybutyric acid (GHB)
d) Flunitrazepam (Rohypnol)
e) Ketamine
14) What percentage of sexual assault reports are false?
a) 2%
b) 10%
c) 15%
d) 20%
e) 50%
15) Which of the following is NOT a common symptom of post-traumatic stress disorder
(PTSD)?
a) Nightmares
b) Depression
c) Jumpiness
d) Euphoria
e) Flashbacks
16) About how long can the disorganization phase of rape-trauma syndrome (RTS) last?
a) 24-hours
b) Four days
c) Two weeks
d) Two months
e) Four years
17) The reorganization phase of RTS can last up to how long?
a) 24-hours
b) Four days
c) Two weeks
d) Two months
e) Four years

DO NOT DISTRIBUTE - 68 -
Human Sexuality 14Mar2009

18) Which of the following is the most important factor that influences recovery after
RTS?
a) Relationship with offender
b) Degree of violence
c) Social influences
d) Cultural influences
e) Immediate personal contacts after the assault
19) In the United States, there is a rape every:
a) Month
b) Day
c) Hour
d) 30-minutes
e) 3-minutes
20) Forensic nurse examiners are registered nurses who are specially trained to provide
comprehensive care to which of the following?
a) Victims of violence
b) Victims of sexual assault
c) Child abuse victims
d) Elder abuse victims
e) All of the above
21) During a sexual assault genital exam, a speculum should not be used if the patient has
not started their menstrual cycle (menarche).
a) True
b) False, a speculum exam should always done
22) Which of the following locations is the most common for genital injury during sexual
assault?
a) Hymen (ecchymosis)
b) Fossa navicularis (tears)
c) Anus (tears)
d) Posterior forchette (tear)
e) Vagina (ecchymosis)
23) An Emergency Room physician would not examine a sexual assault patient’s genital
area unless requested by the forensic nurse examiner (FNE).
a) True
b) False
24) Toluidine Blue Dye, which adheres to nucleated squamae in the deeper layers of the
epidermis, should be applied:
a) To the assault victim’s hands before evidence collection
b) To the assault victim’s hands after evidence collection
c) To the assault victim’s genital area before evidence collection
d) To the assault victim’s genital area after evidence collection
25) Which of the following is tested via blood and not culture?
a) Gonorrhea
b) Chlamydia
c) Syphilis
d) Trichamonis

DO NOT DISTRIBUTE - 69 -
Human Sexuality 14Mar2009

26) Blood samples taken for a suspected drug facilitated sexual assault should be drawn
(gray top blood tube) up to how many hours after an incident?
a) 6
b) 12
c) 24
d) 48
e) 96
27) Date rape drugs may remain detectable in the urine up to how many hours?
a) 6
b) 12
c) 24
d) 48
e) 96
Match the medication with the reason for treatment:
28) Chlamydia a) Ceftriaxone (Rocephin)
29) Pregnancy b) Azithromycin (Zithromax)
30) Gonorrhea c) Levonorgestrel (Plan B)
31) In what situation(s) can physician-patient confidentiality be broken?
a) Never (HIPAA laws)
b) If the patient plans to harm them self
c) If the patient plans to harm others
d) If a minor is being abused
e) B, C, & D

James Lamberg

DO NOT DISTRIBUTE - 70 -
Human Sexuality 14Mar2009

AnswerKey 6.1) C 13.18) A 16.2) C


HS #1 6.2) B 13.19) E 17) A HS #9
1) E 6.3) D 13.20) A 18) B 1.1) A
2.1) B 13.21) C 1.2) A
2.2) D HS #3 13.22) B HS #5 2) D
2.3) A 1.1) A 13.23) D 1) D 3.1) A
2.4) C 1.2) E 13.24) A 2) A 3.2) A
3) C 1.3) D 13.25) D 3) B 4) E
4.1) A 2) C 13.26) B 4) C 5) C
4.2) C 3) E 14.1) D 5.1) A 6) A
4.3) D 4.1) A 14.2) C 5.2) D 7) E
5.1) A 4.2) B 6) C 8.1) A
5.2) E 5) B HS #4 7) C 8.2) B
5.3) A 6.1) A 1.1) D 8) C 8.3) D
5.4) C 6.2) D 1.2) C 9) A 9) A
5.5) D 7.1) C 2) C 10) D 10.1) A
6) E 7.2) A 3.1) B 11) B 10.2) D
7) A 8) E 3.2) E 12) C 11.1) B
8.1) B 9) E 4) D 13) B 11.2) A
8.2) A 10) A 5.1) A 14.1) A 11.3) B
8.3) A 11.1) D 5.2) C 14.2) B 11.4) B
8.4) B 11.2) E 6) E 14.3) A 11.5) B
8.5) A 11.3) A 7) E 14.4) A 11.6) B
8.6) A 12.1) A 8.1) C 14.5) A 11.7) A
8.7) A 12.2) A 8.2) B 15.1) C 11.8) B
8.8) B 12.3) B 9.1) A 15.2) A 11.9) B
9.1) D 12.4) B 9.2) B 16.1) A 12.1) C
9.2) C 12.5) B 10) A 16.2) E 12.2) D
9.3) C 12.6) B 11) C
9.4) E 13.1) A 12.1) D HS #6 HS #10
9.5) D 13.2) C 12.2) E 1.1) D 1) B
9.6) A 13.3) A 12.3) A 1.2) D 2.1) E
13.4) B 13.1) D 2) E 2.2) B
HS #2 13.5) A 13.2) E 3.1) C 3.1) C
1) E 13.6) E 14.1) B 3.2) B 3.2) B
2.1) C 13.7) D 14.2) A 3.3) A 4) D
2.2) D 13.8) B 14.3) A 3.4) A 5.1) A
3) B 13.9) B 14.4) B 4.1) A 5.2) A
4.1) C 13.10) A 14.5) B 4.2) A 5.3) A
4.2) A 13.11) E 14.6) B 4.3) A 6) A
5.1) A 13.12) D 14.7) A 4.4) B 7) C
5.2) A 13.13) C 15.1) C 5.1) C 8) C
5.3) A 13.14) B 15.2) C 5.2) A 9.1) A
5.4) B 13.15) A 15.3) A 5.3) C 9.2) B
5.5) A 13.16) E 15.4) A 5.4) E 10) E
5.6) B 13.17) E 16.1) E 6) A 11.1) B

DO NOT DISTRIBUTE - 71 -
Human Sexuality 14Mar2009

11.2) A 11) D 21.1) A 12.3) B 16.2) B


12) D 12) B 21.2) B 12.4) B 16.3) B
13.1) E 13) C 22) A 12.5) A 16.4) B
13.2) A 14.1) A 23) A 12.6) A 16.5) A
14) B 14.2) A 12.7) A 16.6) A
15.1) B 14.3) B HS #14 13.1) A 16.7) B
15.2) A 14.4) A 1) A 13.2) D 17.1) C
15.3) A 14.5) A 2) D 14.1) A 17.2) D
15.4) A 14.6) B 3) A 14.2) B 17.3) B
15.5) B 14.7) B 4) D 17.4) B
15.6) A 15.1) B 5) A HS #16
16.1) B 15.2) C 6) A 1) A HS #18
16.2) B 15.3) A 7) C 2.1) C 1) A
16.3) D 16.1) A 8) A 2.2) E 2) A
16.2) A 9) D 2.3) A 3) C
HS #11 16.3) A 10) E 3) E 4) A
1) D 11) A 4.1) L 5) A
2) E HS #13 12.1) A 4.2) K 6) A
3.1) E 1) C 12.2) A 4.3) J 7) C
3.2) D 2) B 12.3) A 4.4) I 8) A
4.1) E 3) B 12.4) A 4.5) H 9) E
4.2) B 4.1) B 12.5) B 4.6) G 10.1) A
5) B 4.2) C 12.6) B 4.7) F 10.2) A
6) C 5) A 12.7) B 4.8) E 10.3) B
7.1) A 6) A 12.8) B 4.9) D 10.4) B
7.2) B 7) B 13.1) D 4.10) C 10.5) B
7.3) A 8) E 13.2) A 4.11) B 10.6) A
7.4) A 9) C 4.12) A 11) D
7.5) A 10) C HS #15 5.1) B
7.6) B 11.1) A 1) D 5.2) A HS #19
7.7) B 11.2) B 2.1) A 5.3) E 1) B
7.8) A 12) C 2.2) B 6.1) C 2) E
8) D 13) C 2.3) D 6.2) D 3.1) B
14) C 3) C 6.3) E 3.2) B
HS #12 15) D 4) E 6.4) B 3.3) A
1) E 16) B 5) E 6.5) A 3.4) B
2.1) C 17) A 6) A 7) E 3.5) A
2.2) D 18) A 7.1) B 8) D 3.6) A
3) B 19.1) A 7.2) A 9) F 4.1) B
4) B 19.2) B 8) A 10) C 4.2) A
5) E 19.3) B 9.1) A 11) B
6) E 19.4) B 9.2) D 12) C HS #20
7) D 19.5) A 10) A 13) C 1) E
8) A 19.6) A 11) A 14) B 2) E
9) D 20.1) C 12.1) B 15) B 3) B
10) C 20.2) A 12.2) B 16.1) A 4) B

DO NOT DISTRIBUTE - 72 -
Human Sexuality 14Mar2009

5) D 51) A 97) D 26) C


6) C 52) A 98) A 27) E
7) E 53) C 99) A 28) D
8) A 54) D 100) B 29) C
9) E 55) A 101) A 30) B
10) E 56) C 102) A 31) E
11) D 57) D 103) B 32) E
12) E 58) B 104) C 33) C
13) D 59) E 105) B 34) E
14) D 60) D 106) A 35) D
15) B 61) C 107) E 36) D
16) A 62) A 108) A 37) A
17) E 63) A 109) B 38) E
18) A 64) E 110) B
19) C 65) A 111) D HS #22
20) B 66) C 112) A 1) E
21) E 67) A 113) C 2) B
22) C 68) B 114) E 3) C
23) C 69) B 115) C 4) B
24) C 70) A 5) A
25) D 71) A HS #21 6) E
26) E 72) E 1) D 7) D
27) A 73) C 2) C 8) A
28) D 74) D 3) C 9) F
29) B 75) A 4) A 10) C
30) C 76) E 5) D 11) B
31) E 77) D 6) A 12) C
32) F 78) B 7) D 13) E
33) E 79) C 8) A 14) A
34) D 80) C 9) C 15) D
35) C 81) A 10) B 16) D
36) A 82) D 11) B 17) E
37) C 83) E 12) A 18) E
38) E 84) B 13) C 19) E
39) C 85) D 14) A 20) E
40) D 86) C 15) B 21) A
41) C 87) E 16) B 22) D
42) F 88) E 17) A 23) A
43) A 89) B 18) C 24) D
44) E 90) D 19) A 25) C
45) F 91) A 20) B 26) C
46) D 92) A 21) B 27) E
47) C 93) B 22) C 28) B
48) A 94) B 23) B 29) C
49) E 95) E 24) E 30) A
50) A 96) D 25) C 31) E

DO NOT DISTRIBUTE - 73 -

Você também pode gostar